Sherpath Questions

Ace your homework & exams now with Quizwiz!

Match the category of pregnancy signs and symptoms to the example. Nausea and vomiting; Visualization of fetal outline by ultrasound; Softened uterus Presumptive; Positive, Probable

Visualization of fetal outline by ultrasound: Positive Softened uterus: Probable Nausea and vomiting: Presumptive

Match the reflex being assessed with the picture. Sucking; Tonic neck; Moro; Babinski

1 Babinski 2 Tonic neck 3 Sucking 4 Moro

Place the steps in the order in which they would be applied to perform Leopold maneuvers. Palpate the suprapubic area to confirm presentation; Palpate the uterine fundus; Determine if head is flexed (vertex) or extended (face);Palpate for the fetal back.

1. Palpate the uterine fundus. 2. Palpate for the fetal back. 3. Palpate the suprapubic area to confirm presentation. 4. Determine if head is flexed (vertex) or extended (face). Leopold maneuvers are performed to identify fetal presentation and position and to help the nurse locate the fetal back, which is the area of maximal intensity for the fetal heart rate.

List relevant patient cues in the order the nurse recognizes them in a primipara during labor as the patient progresses from the first stage of labor to the second stage of labor. Cervix is 6 cm dilated; Contractions become closer and coordinated; Patient wakes up and feels mild contractions at home; Cervix is thick and 50% effaced on assessment at the hospital; Patient has bloody show and reports rectal pressure.

1. Patient wakes up and feels mild contractions at home. 2. Cervix is thick and 50% effaced on assessment at the hospital. 3. Contractions become closer and coordinated. 4. Cervix is 6 cm dilated. 5. Patient has bloody show and reports rectal pressure. The relevant patient cues the nurse recognizes as the patient progresses from the first stage of labor to the second stage of labor in order would be feeling mild contractions at home, then the cervix is thick and 50% effaced on assessment at the hospital. Next the contractions become closer and coordinated and then the cervix is 6 cm dilated. And the then the patient has bloody show and reports rectal pressure.

The nurse understands that an early postpartum hemorrhage is defined as an estimated blood loss greater than __ mL in the first 24 hours after a vaginal delivery.

500 mL

A patient in the early postpartum period asks, "How do I know if my baby is getting enough breast milk? I don't feel anything there." What are the desirable outcomes? A) "A good way to tell if your baby is taking in enough breast milk is if you continue to see enough wet and soiled diapers." B) "If your baby is gaining weight with each checkup at the pediatrician, it usually means your baby is getting enough breast milk." C) "If your baby actively participates in a feeding, you can hear swallowing and your baby seems content in between feeding. These are all signs that your baby is satisfied with the quantity of breast milk your body is producing." D) "If your baby feeds for at least 10 minutes on each breast with every feed, you know your baby is getting enough breast milk." E) "When your milk comes in, your breasts will feel full and most women will feel a milk letdown. This is a good sign that your body is producing enough breast milk for your baby."

A) "A good way to tell if your baby is taking in enough breast milk is if you continue to see enough wet and soiled diapers." (Monitoring newborn output is an important component when assessing whether intake is adequate.) B) "If your baby is gaining weight with each checkup at the pediatrician, it usually means your baby is getting enough breast milk." (Adequate weight gain is the single most important consideration when determining whether breastfeeding is successful.) C) "If your baby actively participates in a feeding, you can hear swallowing and your baby seems content in between feeding. These are all signs that your baby is satisfied with the quantity of breast milk your body is producing." (Swallowing and satiation are signs that the newborn is effectively transferring milk from the breast with feeding.) E) "When your milk comes in, your breasts will feel full and most women will feel a milk letdown. This is a good sign that your body is producing enough breast milk for your baby." (Feelings of fullness in the breast and the sensation of letdown occur when mature milk forms in the breasts. These are signs that breastfeeding is going well and that milk supply is likely adequate.)

Which nursing response is appropriate for a perimenopausal woman who states she no longer needs to use birth control because her periods are irregular? A) "Although your periods are irregular, you can still become pregnant." B) "You are right. You are very unlikely to get pregnant." C) "Because your periods are irregular, birth control is not necessary." D) "Are you worried about becoming pregnant?"

A) "Although your periods are irregular, you can still become pregnant." (Although fertility is reduced, women are encouraged to use some method of birth control because pregnancies can occur.)

Which statement by the nurse is the most supportive of the mother during the taking-hold phase? A) "Because I changed the last diaper, let me talk you through changing this diaper." B) "You are tired; I can take care of the baby for you." C) "When you are done feeding the baby, I will burp the baby for you." D) "If you are tired, just have your mother give the infant a bottle instead of breastfeeding."

A) "Because I changed the last diaper, let me talk you through changing this diaper." (The nurse would instruct mothers how to perform tasks and then reinforce teaching.)

The restoration of the endometrium is dependent upon which factor? A) Estrogen stimulation from the ovarian follicles B) Estrogen stimulation from the corpus luteum C) Estrogen stimulation from luteinizing hormone (LH) D) Estrogen stimulation from the hypothalamus

A) Estrogen stimulation from the ovarian follicles (Estrogen stimulation from the ovarian follicles promotes rapid growth and restoration of the endometrium.)

A postpartum patient verbalizes that she is feeling overwhelmed about going back to work full-time after 6 weeks of maternity leave. Which statement by the nurse would be supportive of the mother and her concerns? A) "I know you're overwhelmed, and it's common to feel that way. What kind of support do you have from friends and family?" B) "I'm concerned you feel that way. I think you should be evaluated for postpartum depression." C) "You might want to consider cutting back your hours at work." D) "Lots of mothers feel that way. You'll learn to adjust."

A) "I know you're overwhelmed, and it's common to feel that way. What kind of support do you have from friends and family?" (The nurse is acknowledging the patient's feelings. At the same time, the nurse is assisting the woman in considering alternatives, such as support from friends and family.)

A patient who is positive for human immunodeficiency virus (HIV) is about to be discharged from the hospital with her new infant. Which patient statement indicates that further patient teaching is required? A) "I'm glad I'll still be able to breastfeed my baby." B) "It's still possible that I gave my baby HIV, even though I took all my medication." C) "I will go to the pharmacy right now to fill my antiretroviral drug." D) "Even though I don't feel sick, I should plan on bottle-feeding my baby."

A) "I'm glad I'll still be able to breastfeed my baby." (The patient's statement that she will be able to breastfeed her baby is incorrect, so further teaching is required. Breast milk is a route of HIV virus transmission and should be avoided completely.)

Which statement would a nurse anticipate from a postpartum mother in the letting-go phase? A) "It seems strange not to feel the baby kicking in my uterus." B) "Can you feed the infant a bottle? I need to sleep." C) "Am I putting the diaper on correctly?" D) "I can't believe that the delivery happened so quickly."

A) "It seems strange not to feel the baby kicking in my uterus." (As the mother adjusts to the reality of no longer being pregnant, she would verbalize her feelings about not being pregnant anymore, and the nurse would offer reassurance that these feelings are typical postpartum.)

A primigravida Hispanic patient asks why she needs to be screened for gestational diabetes when she is not a diabetic. Which response by the nurse is appropriate? A) "Many patients with gestational diabetes show no symptoms. It is an important way to help you have a healthy pregnancy." B) "We test all patients for diabetes because today's diet makes you more likely to get diabetes." C) "Your race makes you more likely to develop gestational diabetes, so we are screening you just in case." D) "Because this is your first pregnancy, we don't know what to expect, so we are going to test you just in case."

A) "Many patients with gestational diabetes show no symptoms. It is an important way to help you have a healthy pregnancy." (Most patients with gestational diabetes are asymptomatic. For this reason, universal screening is best practice to promote the best pregnancy outcomes.)

An obese woman reports having no body image-related distress and asks if being obese is "really that bad." Which preventive counseling statements are appropriate? A) "Obesity puts you at risk for multiple types of cancer." B) "The only disease you are at significant risk for is diabetes." C) "If you're planning to have children, you should be aware that obesity puts you at risk for fertility problems, miscarriage, and stillbirth." D) "Inactivity related to obesity puts you at greater risk for stroke." E) "As long as you are comfortable with your body image, there is no reason for you to alter your diet or exercise habits."

A) "Obesity puts you at risk for multiple types of cancer." (Obesity is associated with multiple chronic diseases, including breast, endometrial, and colon cancer.) C) "If you're planning to have children, you should be aware that obesity puts you at risk for fertility problems, miscarriage, and stillbirth." (Gynecologic conditions associated with obesity include abnormal menses and infertility.) D) "Inactivity related to obesity puts you at greater risk for stroke." (Inactivity related to obesity is associated with osteoporosis, osteoarthritis, dyslipidemia, stroke, and coronary artery disease.)

Which recommendations would the nurse make to the family whose newborn falls asleep after 2 to 3 minutes of breastfeeding? A) "The baby needs to be awakened, so unwrap the newborn and gently stroke his palms or soles." B) "This is a sign that the newborn is full, so breastfeeding should stop until the baby shows hunger cues." C) "Change the newborn's diaper to wake your newborn up." D) "The newborn is having trouble establishing breastfeeding, so switch to bottle feeding." E) "This is a sign of nipple confusion, so provide pacifiers between feedings."

A) "The baby needs to be awakened, so unwrap the newborn and gently stroke his palms or soles." (Newborns who fall asleep shortly after beginning the feeding need to be woken up to feed properly. Unwrapping and gently stroking the newborn will help to awaken for feeding.) C) "Change the newborn's diaper to wake your newborn up." (Changing the diaper is a useful strategy for awakening a sleepy newborn for feeding.)

Which education would the nurse provide a patient who is worried that the intrauterine catheter will hurt the baby? A) "The catheter is placed in the uterus next to the baby." B) "Don't be nervous; everything will be fine!" C) "The catheter attaches only to the outer layer of skin on the baby's head." D) "This is the equipment needed for high-risk pregnancies."

A) "The catheter is placed in the uterus next to the baby." (By providing information about the safety and location of the catheter, the nurse can decrease the patient's anxiety and provide reassurance.)

The nurse caring for a patient with a history of a previous cesarean delivery notes a small incision scar on the patient's bikini line (transverse). The patient asks the nurse if a trial of labor will be possible. Which response would the nurse provide? A) "Transverse uterine incisions allow for vaginal births, but the skin and uterine incisions do not always match, so we will have to verify." B) "You have a transverse skin incision, so that means you also have a transverse uterine incision and a trial of labor is possible." C) "Because you have a transverse skin incision, there is a greater chance of having a uterine incision rupture with labor." D) "It is never safe to have a vaginal birth after a cesarean delivery."

A) "Transverse uterine incisions allow for vaginal births, but the skin and uterine incisions do not always match, so we will have to verify." (The nurse needs to verify the type of incision on the uterus because it may not match the skin incision.)

Which factors may affect the sperm quality in a male? A) Untreated gonorrhea B) Nightly hot baths C) Varicocele D) Impotence E) Obesity

A) Untreated gonorrhea (Untreated sexually transmitted infections, such as gonorrhea, may lead to a decrease in sperm quality and quantity.) B) Nightly hot baths (Testicular exposure to high temperatures can negatively affect the quality of sperm.) E) Obesity (Obesity in a man can lead to a decrease in the quality of sperm.)

Which nursing questions are appropriate for a patient 9 days postpartum who feels tired and still has vaginal discharge? A) "What color is your lochia?" B) "Is your lochia red?" C) "Is there an odor to your lochia?" D) "When did your lochia turn clear?" E) "How often are you changing your peripads?"

A) "What color is your lochia?" (Lochia is associated with uterine involution and changes in the endometrium, is described according to color and amount, and should be serosa by 9 days postpartum.) C) "Is there an odor to your lochia?" (Lochia should not have an offensive odor but should smell like normal menstrual flow.) E) "How often are you changing your peripads?" (The volume of lochia should be decreasing 9 days postpartum, so it would be helpful to know how often the woman is changing her peripad to approximate discharge amount.)

A nurse is meeting with a woman to confirm her pregnancy at week 8; a sonogram technician is also present to confirm the fetal heartbeat by ultrasound. The woman says, "I'm so excited to see a little human." Which response would the nurse make to clarify development expectations for this woman? A) "Yes, your baby will definitely look like a tiny human now." B) "You should know the sex too!" C) "Your baby should look like a ball of cells." D) "Your baby will still look like a little tadpole."

A) "Yes, your baby will definitely look like a tiny human now." (This is an appropriate response, as the embryo has a definite human form by week 8.)

When teaching a patient with a body mass index (BMI) of 20 about recently prescribed subcutaneous insulin, which statement is correct? A) "You will need to insert the needle at a 45-degree angle to ensure it is delivered to the subcutaneous tissue." B) "If you need to deliver more than 0.5 mL of insulin, you will need to use two injection sites." C) "If you use the no-pinch method, you should use an 8-mm needle." D) "Gently massage the injection site with sterile gauze after administration to ensure the medication is absorbed."

A) "You will need to insert the needle at a 45-degree angle to ensure it is delivered to the subcutaneous tissue." (This patient has a very low BMI and likely minimal subcutaneous tissue. Entering at a 45-degree angle will ensure the medication is delivered into subcutaneous tissue and not muscle.)

A woman at 8½ weeks gestation presents for a prenatal checkup. She is eager to find out the baby's sex. Which response by the nurse best explains genitourinary changes at this stage? A) "Your baby's external genitalia are evolving, but gender cannot be determined until week 10." B) "The baby's DNA will assign the sex after week 10." C) "At this phase, nothing changes in regard to the genitalia." D) "External genitalia begin differentiating at week 8, so let's take a look."

A) "Your baby's external genitalia are evolving, but gender cannot be determined until week 10." (Although in week 8 the external genitalia begin to differentiate, male and female characteristics are not distinct until after the 10th week.)

Which statements regarding the assessment of newborn vital signs are accurate? A) A heart rate of 146 beats/min is normal. B) A respiratory rate of 66 breaths/min is normal. C) A temperature of 35.8°C (96.4°F) is normal. D) A blood pressure of 70/48 is normal. E) Respiratory rate and heart rate are assessed for a full minute. F) Blood pressure is routinely assessed.

A) A heart rate of 146 beats/min is normal. (The normal newborn heart rate ranges from 120 to 160 beats/min.) C) A temperature of 35.8°C (96.4°F) is normal. (Normal newborn axillary temperature ranges from 36.5 to 37.5°C (97.7 to 99.5°F).) D) A blood pressure of 70/48 is normal. (While not routinely measured, normal newborn blood pressure ranges from systolic 65 to 95 mm Hg and diastolic 30 to 60 mm Hg.) E) Respiratory rate and heart rate are assessed for a full minute. (Respiratory rate and heart rate are routinely assessed for a full minute to account for variability.)

Which patient would benefit from prenatal genetic counseling? A) A patient with sickle cell disease (SCD) B) A patient who is having difficulty conceiving C) A patient with history of two multifetal pregnancies D) A patient with a history of miscarriage resulting from polyploidy

A) A patient with sickle cell disease (SCD) (SCD is an autosomal recessive trait that can severely affect a patient's life. This patient would benefit from genetic counseling at any point in the perinatal care period.)

Which postpartum women may require nursing care for ineffective bladder elimination? A) A woman with third-degree perineal laceration and significant edema B) A woman who received epidural anesthesia during labor C) A woman with a first-degree perineal laceration and slight edema D) A woman with an episiotomy and a firm, midline fundus at the umbilicus E) A woman who delivered by cesarean birth and had an indwelling catheter removed

A) A woman with third-degree perineal laceration and significant edema (A woman with extensive vaginal or perineal lacerations is at risk for ineffective bladder elimination as a result of an altered voiding reflex.) B) A woman who received epidural anesthesia during labor (A woman who received epidural anesthesia is at risk for ineffective bladder elimination as a result of decreased sensation to void and altered voiding reflex.) E) A woman who delivered by cesarean birth and had an indwelling catheter removed (A woman who delivered by cesarean birth and had an indwelling catheter removed would be at risk for ineffective bladder elimination.)

Which intervention would the nurse implement for a 10-minute-old healthy newborn? A) Administer vitamin K. B) Assess blood pressure. C) Obtain serum glucose level. D) Perform heelstick for universal newborn screening.

A) Administer vitamin K. (Newborns are born without clotting factors. Vitamin K is necessary for clotting factor production to prevent bleeding disorders. The nurse would administer the vitamin K first to prevent excessive bleeding.)

A G5/P4 is experiencing painful contractions that start in her back every 10 minutes for the last hour. How would the telephone triage nurse advise this woman? A) Advise the patient to report to the hospital for labor and delivery. B) Tell the patient to call an ambulance. C) Instruct the woman to come to the hospital when the contractions are every 5 minutes. D) Inform the patient that she is experiencing Braxton Hicks contractions.

A) Advise the patient to report to the hospital for labor and delivery. (Multiparas should report to the hospital or birthing center with regular contractions for at least an hour that are 10 minutes apart.)

Which assessments would the nurse include during the initial assessment of a newborn after birth? A) Airway patency B) Heart sounds C) Head circumference D) Length E) Color

A) Airway patency (The nurse would assess the airway patency of the newborn; if the airway is not patent, resuscitation or immediate intervention is needed.) B) Heart sounds (The nurse would assess the heart sounds of the newborn (heart rate, rhythm, and whether murmur is present) to determine whether resuscitation or immediate intervention is needed.) E) Color (The nurse would assess the color of the infant. If not pink, the infant may require resuscitation or immediate intervention.)

Which conditions are significant causes of deep vein thrombosis (DVT)? A) Altered blood coagulation B) Stasis of blood C) High-grade fever D) Positive Homans sign E) Vessel wall injury

A) Altered blood coagulation (The blood clots of DVT can be caused by anything that prevents the blood from circulating or clotting normally. Altered blood coagulation can cause the blood to clot abnormally and create a blood clot.) B) Stasis of blood (The blood clots of DVT can be caused by anything that prevents the blood from circulating or clotting normally. Venous stasis may occur after surgery when a patient is not ambulating. Early ambulation and use of elastic compression socks will help prevent venous stasis.) E) Vessel wall injury (The blood clots of DVT can be caused by anything that prevents the blood from circulating or clotting normally. Any injury or inflammation to the vessel wall can alter blood circulation, which can result in a blood clot.)

Which measures should the nurse ensure are available and ready before a multiple gestation twin delivery? A) An operating room set up for vaginal and cesarean delivery B) Neonatal health care providers capable of advanced resuscitation for each baby C) A single infant warmer with supplies for each infant D) Two separate infant warmers with separate supplies for each baby E) A fetal monitor with the capacity to monitor two babies at the same time

A) An operating room set up for vaginal and cesarean delivery (In the event of an emergency or the need for an urgent cesarean delivery, multiple gestation deliveries are often done in the operating room, even when the planned delivery route is vaginal.) B) Neonatal health care providers capable of advanced resuscitation for each baby (Whether delivery is vaginal or cesarean, the neonatal care staff must be prepared for the care and possible resuscitation of multiple infants. Cord clamps, bulb syringes, radiant warmers, and resuscitation equipment must be prepared for each infant. A neonatal care team of nurses, a neonatal nurse practitioner, and a pediatrician or a neonatologist should be available for each infant, with another nurse caring for the mother.) D) Two separate infant warmers with separate supplies for each baby (Two infant warmers should be available in the event that resuscitation is necessary of one or both babies.) E) A fetal monitor with the capacity to monitor two babies at the same time (During labor, each fetus's FHR is monitored separately. While resting in bed, the patient should be in a lateral position to promote placental blood flow. Assessment of each FHR continues until delivery.)

When cardiovascular disease is severe during pregnancy, which health care providers may be a part of the interdisciplinary care team? A) Anesthesiologists B) Nurses C) Dermatologists D) Neonatologists E) Obstetricians F) Maternal-fetal medicine specialists

A) Anesthesiologists (Anesthesiologists are important team members who help manage patient conditions, administer analgesia in labor, and manage airway and hemodynamic stability in the operating room. They should be involved in care long before delivery.) B) Nurses (Nurses help monitor, care for, and implement interventions.) D) Neonatologists (Neonatologists must be involved in the care of the pregnant patient, as the neonate may have been exposed to dangerous medications, may have growth restrictions, and may have many other complications.) E) Obstetricians (Obstetricians lead the team in the care for the pregnant patient and work closely with specialists to develop a plan of care.) F) Maternal-fetal medicine specialists (Maternal-fetal medicine specialists focus on high-risk pregnancy and implement advanced monitoring and interventions specific to the high-risk pregnancy.)

A woman reports dissatisfaction when using her vaginal hormonal ring and asks about starting another method of contraception. Which steps would the nurse take? A) Ask the woman what qualities she is looking for in a contraceptive method. B) Provide the woman with information about multiple other contraceptive methods. C) Ask the woman what she finds dissatisfying about her current contraceptive method. D) Instruct the woman about proper vaginal ring usage and plan on discussing switching methods at the next visit. E) Inform the woman that learning how to use another contraceptive method will distract her from learning how to use her current method correctly.

A) Ask the woman what qualities she is looking for in a contraceptive method. (Assessing the woman's contraceptive needs helps the provider determine which contraceptive methods may be right for the woman. It increases the efficacy of nursing education.) B) Provide the woman with information about multiple other contraceptive methods. (Providing the woman with information about multiple other methods gives the woman a choice, and it allows the woman to determine which methods she believes would be right for her.) C) Ask the woman what she finds dissatisfying about her current contraceptive method. (Asking what the woman finds dissatisfying about her current method of contraception allows the nurse to determine which methods might be better suited for the woman's needs.)

A woman delivered a 9-lb, 10-oz baby 1 hour ago. When the nurse arrives to perform the 15-minute assessment, the patient says that she "feels all wet underneath." The nurse discovers that both perineal pads are completely saturated and that the patient is lying in a 6-inch-diameter puddle of blood. After calling for help, which action would the nurse take next? A) Assess the fundus for firmness. B) Estimate the blood loss by weighing the perineal pads. C) Check the perineum for lacerations. D) Manually remove any contents in the uterus.

A) Assess the fundus for firmness. (Firmness of the uterus is necessary to control bleeding from the placental site. The nurse should first attempt to firm the fundus through firm, but not vigorous, massage.)

The nurse is assessing a 3-day-postpartum breastfeeding woman. Her breasts are firm and warm to the touch. When asked when she last fed the infant, her reply is, "I fed the baby last evening, but I let the nurses feed him in the nursery last night because I needed to rest." Which action would the nurse take to prevent the woman from developing mastitis? A) Encourage the woman to breastfeed her infant more frequently. B) Have the woman massage her breasts hourly. C) Obtain a prescription to culture her expressed breast milk. D) Take the temperature and pulse rate of the woman.

A) Encourage the woman to breastfeed her infant more frequently. (This woman is experiencing engorgement. To alleviate the engorgement and prevent mastitis, the woman needs to breastfeed or pump her breasts more frequently.)

The nurse caring for a patient in the postanesthesia care unit (PACU) notes that a patient is having bradypnea after general anesthesia. Which immediate interventions are indicated to determine the next actions by the nurse? A) Assess the patient's level of consciousness. B) Ensure the pulse oximeter is on and within normal limits. C) Call the anesthesiologist for possible intubation. D) Administer the prescribed dose of naloxone. E) Continue to monitor the patient, as bradypnea is a normal response to general anesthesia.

A) Assess the patient's level of consciousness. (Assessing the patient's level of consciousness is correct. Sometimes general anesthesia can make a patient sleepy, but by gently waking the patient, the respiratory rate may return to normal.) B) Ensure the pulse oximeter is on and within normal limits. (Whenever the patient's respiratory status is compromised, a pulse oximeter is indicated.)

Which intervention by the health care team will decrease the risk of infection for a patient with preterm premature rupture of membranes during labor, delivery, and the postpartum period? A) Avoiding frequent vaginal examinations to check for cervical dilation B) Inserting an indwelling urinary catheter to measure urinary output C) Checking maternal temperature every 12 hours D) Using clean technique when performing speculum examinations

A) Avoiding frequent vaginal examinations to check for cervical dilation (Avoiding frequent vaginal examinations, once the membranes are ruptured, helps prevent the introduction of pathogenic bacteria to the vagina and uterus, which may infect the mother or fetus.)

A woman has a 6-month history of abnormal Papanicolaou (Pap) tests. Which screening procedures should be used to further examine any persistent abnormalities? A) Biopsy B) Colposcopy C) Pelvic examination D) Bimanual examination E) Vulvar self-examination

A) Biopsy (Biopsy of suspicious lesions should be performed for women with a history of abnormal Pap tests to further examine the abnormal tissue. Further examination helps determine if the abnormal tissue is the result of precancerous growth.) B) Colposcopy (Colposcopy (the examination of vaginal and cervical tissue with a colposcope for cell magnification) should be performed for women with a history of abnormal Pap tests to further examine the abnormal tissue. Further examination helps determine if the abnormal tissue is the result of precancerous growth.)

The nurse is completing a medication reconciliation for a pregnant patient with chronic hypertension. Which medication would cause the nurse to alert the health care provider? A) Captopril (angiotensin-converting enzyme [ACE] inhibitor) 25 mg bid B) Labetalol (beta blocker) 200 mg tid C) Nifedipine (calcium channel blocker) 30 mg daily D) Oral (PO) hydralazine (vasodilator) 25 mg tid

A) Captopril (angiotensin-converting enzyme [ACE] inhibitor) 25 mg bid (ACE inhibitors are contraindicated in pregnancy.)

Which newborn assessment finding is considered a normal variant? A) Caput succedaneum B) Peripheral acrocyanosis C) Minimal tearing D) Nostrils patent bilaterally

A) Caput succedaneum (Caput succedaneum is an area of edema to the newborn's head as a result of labor; it is considered a normal variant.)

Which characteristics are associated with sickle cell anemia? A) Causes frequent hospitalization throughout the life span B) Characterized by distorted and decreased erythrocytes C) Can lead to preterm birth D) Causes severe pain E) Commonly leads to fetal neural tube defects F) May be cured with diet and supplementation

A) Causes frequent hospitalization throughout the life span (Crises may cause frequent hospitalization from childhood and beyond.) B) Characterized by distorted and decreased erythrocytes (Erythrocytes appear sickled under microscopy.) C) Can lead to preterm birth (Pregnant women with sickle cell anemia are at increased risk for preterm birth.) D) Causes severe pain (Sickle cell patients have extreme pain when experiencing a flare-up.)

While evaluating an external monitor tracing of a woman who just received regional analgesia for pain management, the nurse notes recurrent late decelerations. Which action should the nurse take next? A) Change the woman's position. B) Notify the obstetric provider. C) Assist with amnioinfusion. D) Rupture the membranes and insert a scalp electrode.

A) Change the woman's position. (Late decelerations may be caused by maternal supine hypotension syndrome. They may be corrected when the woman turns on her side to displace the weight of the gravid uterus from the vena cava.)

Which risk factor places a woman at risk for subinvolution of the uterus? A) Chorioamnionitis B) Forceful traction of the umbilical cord C) Vacuum extraction D) Precipitous delivery

A) Chorioamnionitis (Subinvolution of the uterus is the delayed return (longer than 24 hours) of the uterus to normal size after delivery. Pelvic infections, including chorioamnionitis, place a woman at risk for subinvolution.)

Which medication is indicated for ovulation induction? A) Clomiphene citrate B) Follitropins C) Exogenous progesterone D) Metoclopramide

A) Clomiphene citrate (Clomiphene citrate is an ovulation induction medication used to treat luteal phase inadequacy.)

The antepartum nurse is caring for a patient with a family history of neural tube defects. The patient declines genetic counseling after listening to the health care provider's explanation. Which nursing intervention is appropriate for this patient? A) Continue to the next part of the visit. B) Inform the patient that declining genetic counseling may lead to inadequate preparation if the infant is born with a defect. C) Provide the patient with printed material regarding genetic counseling to take home should the patient have a change of mind. D) Explain to the patient that the test must be done because a family history of neural tube defects indicates a need for genetic counseling.

A) Continue to the next part of the visit. (Whether to receive genetic counseling is ultimately the patient's decision. The nurse would acknowledge the patient's choice and continue with the visit.)

Which fetal condition can cause a fetal heart rate (FHR) of <60 beats/min for more than 60 seconds that does not quickly return to baseline? A) Cord compression B) Fetal head compression C) Uteroplacental insufficiency D) No compromise; variable decelerations are normal

A) Cord compression (Variable decelerations that last a long time and do not quickly return to baseline can occur with cord compression, which could compromise the fetus.)

Which factors could improve a woman's experience with pain during labor? A) Cultural factors B) Financial factors C) Environmental factors D) Psychological factors E) Physiologic factors

A) Cultural factors (Cultural influences may impose certain behavioral expectations regarding acceptable and unacceptable behavior when experiencing pain. Women with strong religious beliefs, for example, often accept pain as a necessary and inevitable part of bringing a new life into the world, whereas others tend to vocalize their pain by moaning, breathing rhythmically, or shouting.) C) Environmental factors (The quality of a laboring woman's environment can play an integral part in how she experiences pain in labor. Environmental factors in labor may include a support person or people present, a supportive physical environment (availability of birthing ball, comfortable chairs, etc.), place of birth (home, birth center, hospital, etc.), and the laboring woman's comfort with her health care providers.) D) Psychological factors (Psychological factors including the laboring woman's partner or support person play a key role in maintaining a calm and supportive environment. Previous experience with childbirth, whether her own or experiencing someone else's birth, can affect the psyche of the laboring woman.) E) Physiologic factors (Physiologic factors, such as understanding the different types of pain, can improve a woman's experience in labor.)

When teaching a childbirth class for first-time parents, the nurse includes which benefits of breathing techniques used during labor? A) Decreases sensation of pain B) Requires increased energy with continued use C) Provides a different focus during contractions D) Can also substitute for pharmacologic therapies E) Can be used with other nonpharmacologic techniques

A) Decreases sensation of pain (The sensation of pain is decreased with breathing techniques.) C) Provides a different focus during contractions (Breathing techniques provide laboring women with a different focus during contractions, reducing the perception of pain. This is a benefit of using breathing techniques in labor.) D) Can also substitute for pharmacologic therapies (Breathing techniques often decrease the need for pharmacologic therapies during labor, as they can work to decrease the sensation of pain.) E) Can be used with other nonpharmacologic techniques (Breathing techniques can be used to supplement other relaxation techniques during labor, such as mental stimulation and cutaneous stimulation.)

Which steps would the nurse take to facilitate effective discussion with a 15-year-old girl who is curious about methods of contraception but is hesitant to ask questions? A) Demonstrate proper contraceptive use to the adolescent. B) Reassure the adolescent that the discussion is confidential. C) Provide the adolescent with written material about different methods of contraception. D) Ask the adolescent about what she has learned about contraception from her peers. E) Inform the adolescent that a discussion about contraception cannot occur without a parent present.

A) Demonstrate proper contraceptive use to the adolescent. (Demonstration can help the adolescent better understand methods of contraception, which in turn may help her use her chosen method of contraception more effectively.) B) Reassure the adolescent that the discussion is confidential. (Adolescents often do not discuss contraception with health care providers because they do not want anyone to know they are sexually active. Reassurance of confidentiality can facilitate better communication between the nurse and the adolescent.) C) Provide the adolescent with written material about different methods of contraception. (Providing written material can help the adolescent by giving her an accurate reference to take home. This can help the adolescent better understand contraception and use it more effectively.) D) Ask the adolescent about what she has learned about contraception from her peers. (Asking the adolescent what she has learned from her peers about contraception is an important step in facilitating education. Adolescents frequently have erroneous beliefs about contraception and determining where the adolescent may lack accurate knowledge helps the nurse effectively educate her about effective contraceptive use.)

Which instructions would the nurse include when educating the family of a patient admitted for preeclampsia? A) Dim the lights. B) Limit activity. C) Increase fluid intake. D) Turn off the television. E) Limit the number of visitors. F) Decrease protein intake.

A) Dim the lights. (Dimming the lights decreases environmental stimuli for preeclamptic patients, which may prevent further blood pressure elevation.) B) Limit activity. (Patients with preeclampsia are placed on modified bed rest or complete bed rest to prevent further blood pressure elevation.) D) Turn off the television. (Turning off the television is important to decrease environmental stimuli for preeclamptic patients.) E) Limit the number of visitors. (Limiting visitors decreases environmental stimuli for preeclamptic patients, which may prevent further blood pressure elevation.)

A G1/P0 patient arrives for elective induction of labor at 39 weeks and is 1 to 2 cm dilated and 50% effaced. The fetal station is −3 with a posterior and firm cervix. Which order is most appropriate based on this patient's Bishop score? A) Discharge to home with follow-up in 1 week at the office. B) Administer oxytocin and titrate per protocol. C) Assist with amniotomy and initiate oxytocin as prescribed. D) Initiate electronic fetal monitoring and insert peripheral intravenous device.

A) Discharge to home with follow-up in 1 week at the office. (This patient has a very low Bishop score, is a primigravida, and is only at 39 weeks gestation. The most appropriate action would be to reevaluate in 1 week because her risk for cesarean delivery is increased based on her Bishop score.)

Which nursing actions are appropriate when the fetal monitor shows a pattern of late decelerations? A) Discontinue oxytocin. B) Provide water to the patient. C) Administer oxygen by face mask. D) Reposition the patient onto her side. E) Continue monitoring and document findings.

A) Discontinue oxytocin. (Discontinuing oxytocin will decrease the effect of uterine stimulants on the contractions.) C) Administer oxygen by face mask. (Oxygen increases the oxygenation to the patient, which increases perfusion of oxygen to the placenta.) D) Reposition the patient onto her side. (Repositioning the patient on her side, rather than allowing a supine position, is preferred. A supine position increases pressure on the vena cava, which reduces the blood supply, causing decreased perfusion of the placenta.)

Which nursing education is appropriate to provide to a pregnant patient reporting hyperemia? A) Encourage the woman to brush her teeth carefully. B) Advise the woman to have her blood pressure checked regularly. C) Encourage the woman to wear supportive hosiery. D) Advise the woman not to lie flat on her back.

A) Encourage the woman to brush her teeth carefully. (Softening and bleeding of the gums can occur due to increased estrogen levels and may be exacerbated by aggressive tooth brushing.)

While monitoring a patient receiving oxytocin for augmentation of labor, the nurse notes tachysystole with recurrent late decelerations and minimal variability on the electronic fetal monitor. Which actions are appropriate? A) Discontinue the oxytocin infusion. B) Reposition the patient on her side. C) Administer an intravenous bolus of fluid per protocol. D) Administer 100% oxygen via tight face mask. E) Notify the health care provider. F) Place the patient in semi-Fowler position and continue to monitor.

A) Discontinue the oxytocin infusion. (Oxytocin infusion can cause tachysystole and decreased uteroplacental perfusion. It should be discontinued when fetal compromise is noted with tachysystole.) B) Reposition the patient on her side. (Repositioning the patient on her side increases uteroplacental perfusion.) C) Administer an intravenous bolus of fluid per protocol. (Administering a fluid bolus is part of the intrauterine resuscitation protocol and is an appropriate action.) D) Administer 100% oxygen via tight face mask. (Administering oxygen is part of the intrauterine resuscitation protocol and is an appropriate action.) E) Notify the health care provider. (The patient is experiencing a medical emergency, and the health care provider should be notified.)

Which effects can a full bladder have on the uterus in the postpartum period? Correct A) Displaces the uterus B) Increases uterine tone C) Promotes a boggy uterus D) Inhibits uterine involution E) Increases uterine involution

A) Displaces the uterus (The uterus is displaced to the right of the umbilicus when the bladder is full.) C) Promotes a boggy uterus (A boggy uterus is associated with a full bladder, which leads to increased blood loss.) D) Inhibits uterine involution (The uterus is unable to contract and involute when the bladder is full.)

Which disorders have a known chromosomal inheritance pattern? A) Down Syndrome B) Turner Syndrome C) Marfan Syndrome D) Fragile X Syndrome E) Edwards Syndrome

A) Down Syndrome (Examples of chromosomal abnormalities include Down, Turner, and Edwards syndromes.) B) Turner Syndrome (Examples of chromosomal abnormalities include Down, Turner, and Edwards syndromes.) E) Edwards Syndrome (Examples of chromosomal abnormalities include Down, Turner, and Edwards syndromes.)

Which intervention can prevent nausea and vomiting in pregnancy? A) Eat small, frequent meals throughout the day. B) Eat one meal per day until the morning sickness subsides. C) Fast food is okay throughout the pregnancy as long as dairy is avoided. D) Drink plenty of fluids, especially carbonated beverages, until the morning sickness subsides.

A) Eat small, frequent meals throughout the day. (An empty stomach can trigger or aggravate nausea, and it may occur on awakening or anytime a person goes hours without food. Grazing on nutritious, small meals throughout the day keeps the stomach satisfied and blood sugar steady.)

The nurse admitting a patient for preeclampsia notes that the patient's platelet count is 95,000. Which additional laboratory result would support the diagnosis of HELLP syndrome? A) Elevated AST/ALT B) Low hemoglobin C) Increased glomerular filtration rate (GFR) D) Decreased serum lactate (LD)

A) Elevated AST/ALT (Elevated liver enzymes AST/ALT are suggestive of the development of HELLP syndrome.)

Which preventive intervention would the nurse implement for a newborn with numerous hyperbilirubinemia risk factors? A) Encourage early feeding. B) Collect total serum bilirubin (TSB). C) Initiate phototherapy. D) Prepare for exchange transfusion.

A) Encourage early feeding. (Feeding stimulates the passage of bilirubin in stools; the nurse would encourage early feeding to promote bilirubin excretion.)

Which approaches would the nurse suggest to new parents to promote adaptation? A) Encourage the parents to delay visitors. B) Emphasize the importance of a strict schedule. C) Encourage the parents to accomplish household tasks when the baby sleeps. D) Encourage the parents to enlist family and friends to help with household tasks. E) Emphasize that the priority during the first 4 to 6 weeks should be caring for themselves and the baby.

A) Encourage the parents to delay visitors. (Encouraging the parents to delay visitors will promote rest.) D) Encourage the parents to enlist family and friends to help with household tasks. (Encouraging the parents to enlist the help of family and friends for household tasks will promote rest.) E) Emphasize that the priority during the first 4 to 6 weeks should be caring for themselves and the baby. (Emphasizing that the priority during the first 4 to 6 weeks should be caring for themselves and the baby will promote rest.)

A G4/P3 patient experiencing precipitate labor presents to the labor wing fully dilated and at +1 station stating that she feels a strong, involuntary urge to push. Which immediate intervention would the nurse take? A) Encourage the patient to push in a side-lying position. B) Initiate an oxytocin infusion to prevent postpartum hemorrhage. C) Assess for umbilical cord prolapse. D) Allow the fetus to rest and descend until birth is imminent.

A) Encourage the patient to push in a side-lying position. (Birth is imminent, and the nurse is responsible for assisting with a safe delivery. A side-lying position enhances placental blood flow and reduces the effects of aortocaval compression associated with a supine position. Side lying slows the rapid fetal descent and minimizes perineal tears.)

A woman with an epidural is 10 cm dilated and the baby is at 0 station. There is a reassuring fetal heart pattern, although the woman does not feel the urge to push. Which are appropriate actions by the nurse? A) Encourage the patient to wait to push until she feels the urge. B) Perform another vaginal examination to check for dilation. C) Encourage the patient to push so that the baby can be born. D) Encourage frequent position changes while remaining in the bed. E) Turn off the epidural so the patient can feel the urge to push.

A) Encourage the patient to wait to push until she feels the urge. (Even with an epidural, a patient usually feels the urge to push as the fetal head descends. This is known as the Ferguson reflex. She should be encouraged to "labor down.") D) Encourage frequent position changes while remaining in the bed. (Frequent position changes help the fetus to descend in the pelvis. This will help trigger the Ferguson reflex. It also promotes uteroplacental perfusion.)

Which statement is consistent with the diagnosis of endometriosis? A) Endometriosis may cause infertility. B) Endometriosis is associated with a higher incidence of cervical cancer. C) Endometriosis can be diagnosed with a blood test. D) Endometriosis is concentrated in the uterus.

A) Endometriosis may cause infertility. (Symptoms of endometriosis may include diarrhea, severe pelvic pain, severe abdominal pain, heavy menstrual bleeding, abnormal bleeding outside of menstruation, and infertility.)

Which factors contribute to the severity of withdrawal symptoms in neonatal abstinence syndrome? Correct A) Exposure amount B) Specific opioid exposure C) Exposure duration D) Exposure timing E) Exposure route

A) Exposure amount (The exposure amount is associated with the severity of withdrawal symptoms in neonatal abstinence syndrome; greater exposure leads to worsened withdrawal symptoms.) C) Exposure duration (The duration of exposure is associated with the severity of withdrawal symptoms in neonatal abstinence syndrome; longer duration leads to worsened withdrawal symptoms.) D) Exposure timing (The timing of exposure is associated with the severity of withdrawal symptoms in neonatal abstinence syndrome; exposure close to birth increases the severity of withdrawal but delays the onset of symptoms.)

The nurse understands that which physiologic changes in the thyroid are considered abnormal in pregnancy? A) Fatigue, edema, and muscle aches may be the result of thyroid activity in pregnancy. B) The thyroid gland enlarges and becomes palpable. C) Thyroid-stimulating hormone (TSH) decreases in the first trimester. D) Free thyroxine (T4) levels increase. E) Increased free thyroxine (T4) levels lead to weight loss and goiter.

A) Fatigue, edema, and muscle aches may be the result of thyroid activity in pregnancy. (Fatigue, edema, and muscle aches may indicate hypothyroidism and are not normal.) B) The thyroid gland enlarges and becomes palpable. (The thyroid gland enlarging and becoming palpable is not an expected finding in pregnancy.) E) Increased free thyroxine (T4) levels lead to weight loss and goiter. (Weight loss and goiter would be abnormal and indicative of hyperthyroidism.)

Which health care provider prescription would the nurse anticipate for a newborn whose heelstick reveals a glucose reading of 49 mg/dL? A) Feed the newborn breast milk or formula. B) Offer glucose water. C) Administer intravenous (IV) fluids. D) Reassess blood glucose in 30 minutes.

A) Feed the newborn breast milk or formula. (A serum glucose of 49 mg/dL is considered hypoglycemia in most agencies; feeding the newborn is the most common intervention for stable hypoglycemia.)

Which assessments may be performed using ultrasonography? A) Fetal anatomy B) Fetal Rh status C) Gestational age D) Chromosomal disorders E) Fetal heart activity F) Amniotic fluid index

A) Fetal anatomy (Ultrasounds are frequently used to assess fetal anatomic structures for abnormality.) C) Gestational age (Gestational age can be estimated with ultrasound by collecting specific measurements in different trimesters.) D) Chromosomal disorders (Certain chromosomal abnormalities may be initially detected using ultrasound soft markers. These findings are not diagnostic, however, and require further workup.) E) Fetal heart activity (Fetal heart activity is easily detected using ultrasonography.) F) Amniotic fluid index (Amniotic fluid amount may be measured using ultrasound.)

A woman is visibly anxious, refuses position changes in labor, and verbalizes, "I'm afraid I'm going to die." The nurse knows it is important to help the woman relax through effective coaching because of which responses to stress experienced in labor? A) Fetal intolerance to labor B) Bradycardia related to a vagal response C) Release of catecholamines in the mother's body D) Inhibited uterine contractions because of decreased blood flow to the placenta E) Umbilical cord prolapse

A) Fetal intolerance to labor (Stress results in the release of catecholamines, which decrease blood flow to the placenta and fetus and can lead to fetal intolerance to labor.) C) Release of catecholamines in the mother's body (Stress in labor results in the release of catecholamines.) D) Inhibited uterine contractions because of decreased blood flow to the placenta (Decreased blood flow to the uterus can lead to inhibited uterine contractions.)

Which supplement would the nurse recommend to prevent fetal neural tube defects as a result of pernicious anemia? A) Folic acid B) Calcium carbonate C) Magnesium hydroxide D) Red raspberry leaf tea

A) Folic acid (Folic acid supplementation has been shown to decrease the incidence of neural tube defects when initiated at least 1 month before conception and continued throughout the pregnancy.)

Which reason explains why adequate amounts of folic acid are needed during pregnancy? A) Folic acid prevents birth defects. B) Folic acid prevents preterm labor. C) Folic acid increases iron absorption. D) Folic acid helps maintain the pregnancy.

A) Folic acid prevents birth defects. (It is important for a pregnant patient to take adequate folic acid to prevent neural tube defects specific to the spinal cord and the brain.)

Which statements are true regarding hyperglycemia? A) Glucose levels in the blood may be elevated, but at the cellular level glucose may be low with some forms of diabetes. B) In gestational diabetes, hyperglycemia always leads to polydipsia. C) The body attempts to dilute the glucose load, resulting in polyuria. D) Glycosuria can occur. E) Glucose is stored in the pancreas when hyperglycemia occurs.

A) Glucose levels in the blood may be elevated, but at the cellular level glucose may be low with some forms of diabetes. (Glucose may be low at the cellular level with hyperglycemia when there is insulin resistance or decreased insulin production.) C) The body attempts to dilute the glucose load, resulting in polyuria. (With hyperglycemia, the body will try to compensate for the high glucose level using the renal system.) D) Glycosuria can occur. (As the body compensates for hyperglycemia, more glucose may be spilled into the urine for excretion.)

The nurse is responsible for intake history at an obstetrics and gynecology (OB/GYN) office. Which medical conditions would the nurse recognize as high-risk conditions that could have adverse outcomes on a pregnancy? A) Hemophilia B) Type 1 diabetes mellitus C) Human immunodeficiency virus (HIV) D) Chronic hypertension E) Xeroderma

A) Hemophilia (Hemophilia is a coagulopathy that increases a woman's risk in pregnancy.) B) Type 1 diabetes mellitus (Type 1 diabetes mellitus is a medical condition that exists before pregnancy and can have serious adverse health outcomes for the pregnant mother and fetus.) C) Human immunodeficiency virus (HIV) (HIV is a medical diagnosis that can have adverse outcomes on a pregnancy.) D) Chronic hypertension (Chronic hypertension can cause adverse outcomes for a mother and fetus during pregnancy.)

Which viral infection can result in neonatal brain complications? A) Herpes infection B) Rubella infection C) Varicella-zoster infection D) Cytomegalovirus infection

A) Herpes infection (A herpes infection can cause neonatal encephalitis.)

Which advantages are associated with human milk (expressed and/or donor milk) feeding? A) Improved infant immunity B) Lowest infant feeding-related costs C) Ability to track amount of milk consumed D) Easy storage and shelf life E) Others able to feed infant F) Promotes maternal-infant attachment

A) Improved infant immunity (Improved infant immunity is an advantage of human milk feeding and exclusive breastfeeding.) C) Ability to track amount of milk consumed (Human milk feeding allows for the tracking of the amount of milk consumed because the infant is fed expressed breast milk from a bottle.) E) Others able to feed infant (An advantage of human milk feeding is that others are able to feed the infant because the infant receives expressed breast milk.)

Which physiologic change in a pregnant woman is associated with nasal congestion? A) Increased estrogen levels B) Increased oxytocin levels C) Increased prolactin levels D) Increased progesterone levels

A) Increased estrogen levels (Increased estrogen levels are associated with increased vascularity to the mucous membranes of the upper respiratory tract during pregnancy. This increases incidence of congestion.)

Which care would the nurse anticipate providing an infant with a brachial plexus injury? A) Intermittent immobilization B) Managing seizures C) Range-of-motion exercises D) Mechanical ventilation E) Gentle manipulation

A) Intermittent immobilization (Intermittent immobilization is used to minimize discomfort and allow for healing in an infant with a brachial plexus injury.) C) Range-of-motion exercises (Range-of-motion exercises are performed to promote range of motion in an infant with a brachial plexus injury, but they are typically delayed until 1 week after the injury to prevent additional injury.) E) Gentle manipulation (Gentle manipulation is indicated to prevent pain for the infant with brachial plexus injury.)

While caring for a formula-fed 2-day-old, which nursing interventions promote safe sleep? A) Keeping bassinet free of toys and supplies B) Placing the newborn prone to sleep C) Offering a pacifier during sleep D) Encouraging the family to bed-share E) Dressing the newborn in a sleep sack

A) Keeping bassinet free of toys and supplies (To avoid suffocation, the bassinet should be free of blankets, pillows, toys, and supplies.) C) Offering a pacifier during sleep (Pacifier use may decrease SIDS, though use may be delayed for the first month until breastfeeding is established.) E) Dressing the newborn in a sleep sack (Dressing the newborn in a sleep sack or one-piece sleeper is recommended to maintain warmth. Blankets and quilts should be avoided due to the risk for suffocation.)

Which duties are appropriate for a support person during labor? A) Keeping patient calm B) Keeping track of contractions C) Interpreting fetal monitoring D) Administering pain medication E) Ensuring physical and emotional comfort

A) Keeping patient calm (The support person should help with breathing and relaxation techniques during labor to keep the patient calm and focused.) B) Keeping track of contractions (The support person should help time contractions during labor. This will help the woman know when to come to the hospital for labor and birth.) E) Ensuring physical and emotional comfort (The support person should help with comfort measures during labor and assist with emotional support of the patient. This will help the woman have less anxiety and promote relaxation during labor.)

A woman in labor has been having regular contractions but has remained 5 cm dilated for 5 hours, with a reassuring fetal heart rate. Which intervention may be necessary for this patient? A) Labor augmentation B) Cesarean delivery C) Vacuum-assisted delivery D) Intrauterine resuscitation

A) Labor augmentation (Labor augmentation may be necessary when labor progress has stopped or slowed.)

Which effect does secretion of human chorionic gonadotropin have on the endometrial cycle of the female reproductive cycle? A) Lack of menstruation B) Release of multiple ova C) Necrosis of the endometrium D) Decreased levels of follicle-stimulating hormone (FSH)

A) Lack of menstruation (Human chorionic gonadotropin is secreted from a fertilized ovum and results in maintenance of the corpus luteum and lack of menstruation during the endometrial cycle.)

Which characteristics predispose the newborn to heat loss? A) Large body surface to mass ratio B) Thin subcutaneous fat layer C) Flexion of extremities D) Maternal temperature at birth E) Superficial blood vessels

A) Large body surface to mass ratio (Newborns have a large skin surface in proportion to their size, increasing the risk for heat loss.) B) Thin subcutaneous fat layer (Newborns have inadequate stores of brown fat and are at a higher risk for heat loss due to thin layers of subcutaneous fat.) E) Superficial blood vessels (Because of the lack of subcutaneous tissue, newborn blood vessels are closer to the surface, leading to increased insensible heat loss.)

Which criterion is used to classify an infant as small-for-gestational-age (SGA)? A) Less than 10th percentile for weight B) 10th percentile for height C) Less than 37 weeks' gestation D) Less than 25th percentile for weight

A) Less than 10th percentile for weight (SGA infants are less than the 10th percentile for weight.)

Which advantages of a low transverse skin incision would a nurse explain to a patient? A) Less visibility when healed B) Less chance of wound dehiscence C) Pubic hair will grow back D) Works well with obese patients E) Allows the surgeon to extend the incision upward F) Quicker to perform

A) Less visibility when healed (The low transverse incision is less visible because it sits at or below the bikini line.) B) Less chance of wound dehiscence (There is less chance of wound dehiscence with low transverse skin incisions.) C) Pubic hair will grow back (Pubic hair will grow back with a low transverse incision. This helps hide the scar.)

A 54-year-old woman with a 20-year history of smoking and a family history of myocardial infarction reports being "prediabetic" and does not exercise regularly. Which screening procedure would the nurse identify as a priority for this woman? A) Lipid profile B) Mammography C) Bone density scan D) Fasting glucose test

A) Lipid profile (A lipid profile would be the priority for this woman as a result of her history of smoking, risk for diabetes, and family history of coronary artery disease (CAD). A high lipid concentration in the blood puts the woman at greater risk for developing CAD, atherosclerosis, and other diseases.)

Which method of anesthesia in labor is considered safest for the fetus? A) Local infiltration B) Epidural block C) Spinal block D) Combined spinal-epidural (CSE) block

A) Local infiltration (Local anesthesia rarely has any adverse effects on either the laboring woman or fetus.)

A patient asks the nurse to describe which factors are included in HELLP syndrome. Which factors would the nurse include in response? A) Low platelets B) Hemolysis C) Low leukocytes D) Elevated erythrocytes E) Elevated liver enzymes

A) Low platelets (Decreased platelet levels are associated with individuals diagnosed with HELLP and define LP in the HELLP mnemonic.) B) Hemolysis (Hemolysis develops as red blood cells pass through blood vessels with vasospasm.) E) Elevated liver enzymes (Liver enzyme levels increase in individuals diagnosed with HELLP and define EL in the HELLP mnemonic.)

Which situations are considered risk factors for a postpartum mood disorder when they are present before the pregnancy? A) Low socioeconomic status B) Strong social support C) Infertility treatments D) History of depression E) 27 years old at time of conception

A) Low socioeconomic status (Women who have a lower socioeconomic status have been found to have a higher prevalence of postpartum depression.) C) Infertility treatments (Women who have gone through the infertility process and required infertility treatment are at an increased risk for postpartum depression due to the stress that the treatments cause before and after conception.) D) History of depression (Women who have had a previous diagnosis of a mood disturbance, such as depression, are at an increased risk for postpartum depression due to the history of a mood disorder.)

A nurse is on the phone with a woman 3 weeks after her intrauterine device (IUD) insertion. Which symptom should be reported to the woman's health care provider immediately? A) Malodorous vaginal discharge B) Strings that can be felt during intercourse C) Stronger cramps during menstruation D) Strings that are the same length as the day of insertion

A) Malodorous vaginal discharge (Unusual or malodorous vaginal discharge is a sign of infection and may indicate serious complications. Women with an IUD are at greater risk for pelvic inflammatory disease, and symptoms of infection should be reported to the health care provider immediately.)

Which major concern regarding epilepsy in pregnancy would the nurse recognize? A) Many antiseizure medications are known teratogens. B) There is an increased risk for miscarriage and preeclampsia. C) A woman who has been seizure-free for many years may have a severe exacerbation in pregnancy. D) Epilepsy in pregnancy may lead to Graves disease.

A) Many antiseizure medications are known teratogens. (Women who are pregnant and have epilepsy may require specialized medication management to prevent adverse fetal effects while managing their condition.)

A laboring patient reports moderate back pain. Her partner rubs her lower back and reminds her to look at the teddy bear they bought for the newborn. Which relaxation techniques are being used? A) Massage B) Focal point C) Hydrotherapy D) Acupressure E) Mental imagery

A) Massage (Massage is a form of cutaneous stimulation that involves rubbing areas of pain to reduce discomfort. This relaxation technique can help with lower back pain during labor.) B) Focal point (The use of a teddy bear to look at during labor is an example of a focal point. By using a focal point, the woman focuses on an external focal point (usually an object with positive associations) as opposed to the internal sensation of pain.)

Which assessment is most important to monitor after the adminiterm-94stration of epidural anesthesia? A) Maternal blood pressure (BP) B) Fetal BP C) Platelet count D) Respiratory rate

A) Maternal blood pressure (BP) (Maternal hypotension may occur after administration of epidural anesthesia; therefore, the nurse must have an accurate maternal BP reading for several minutes after the epidural is placed.)

Which factors are associated with the development of complications in infants of mothers with diabetes? A) Metabolic control before conception B) Gestational age of newborn C) Severity of mother's diabetes D) Age of diabetes onset E) Degree of maternal neurologic complications

A) Metabolic control before conception (Metabolic control before conception and throughout pregnancy is a risk factor contributing to the development of complications in infants of mothers with diabetes; mothers with poor control are more likely to have infants with complications.) C) Severity of mother's diabetes (Infants of mothers with more severe diabetes have higher rates of complications, including congenital anomalies, macrosomia, hypoglycemia, and respiratory distress syndrome.) D) Age of diabetes onset (Mothers with a longer duration of diabetes before pregnancy are more likely to have infants with complications.)

The nurse is caring for a woman in the second stage of labor whose contractions have become more intense. What actions could the nurse take to assist this woman? A) Model modified-paced breathing. B) Model slow-paced breathing. C) Monitor for hyperventilation. D) Conduct acupuncture. E) Ask the patient if it is OK to hold her hand.

A) Model modified-paced breathing. (Modified-paced breathing is most effective in the second stage of labor and involves the pant-blow method of breathing.) C) Monitor for hyperventilation. (Measures to combat hyperventilation include breathing into cupped hands or a paper bag or holding the breath for a couple of seconds. All of these techniques decrease partial pressure of carbon dioxide (PCO2).) E) Ask the patient if it is OK to hold her hand. (Holding a woman's hand is a nonpharmacologic method of pain relief and may help calm the woman during intense contractions. The nurse would always ask the woman's permission before grasping her hand.)

Which nursing intervention would be included in the care of a woman with lupus during her first trimester of pregnancy? A) Monitor for liver function. B) Monitor for increased joint pain and fatigue. C) Provide patient education on remission of lupus. D) Administer subcutaneous heparin for thrombosis prevention.

A) Monitor for liver function. (Renal function, not liver function, often worsens with lupus.)

A patient presents with bleeding caused by placenta previa in late pregnancy. Which physical assessments are the most important for the nurse to perform? A) Monitor urinary output. B) Monitor vaginal bleeding. C) Assess vital signs for the patient. D) Monitor fetal heart rate and contraction patterns. E) Perform digital vaginal examination for dilation. F) Measure fundal height.

A) Monitor urinary output. (Monitoring urinary output is a priority assessment for the nurse. Decreased urine output is a sign of hemorrhage.) B) Monitor vaginal bleeding. (Monitoring vaginal bleeding is a priority assessment for the nurse. Increasing vaginal bleeding or large blood clots may mean hemorrhage.) C) Assess vital signs for the patient. (Assessing vital signs for the patient is a priority assessment for the nurse. Tachycardia and hypotension may exist if bleeding is extensive.) D) Monitor fetal heart rate and contraction patterns. (Monitoring fetal heart rate and contraction patterns are priority assessments for the patient in late pregnancy.)

Which statements regarding sperm preparation for conception are accurate? A) Most sperm are able to fertilize an egg for only 24 hours after ejaculation. B) Sperm obtain fructose for energy from semen. C) Most sperm do not survive more than 24 hours in the female reproductive tract. D) Sperm are protected by the alkaline pH of the semen. E) Sperm immediately fertilize the ovum after ejaculation.

A) Most sperm are able to fertilize an egg for only 24 hours after ejaculation. (Although the sperm can remain viable for 48 hours or more, few sperm have the potential to fertilize an egg for more than 24 hours.) B) Sperm obtain fructose for energy from semen. (Sperm become fully mobile after entering the female reproductive tract. The semen provides fructose for energy and the alkaline pH provides protection from the acidic environment of the vagina.) D) Sperm are protected by the alkaline pH of the semen. (Sperm become fully mobile after entering the female reproductive tract. The semen provides fructose for energy and the alkaline pH provides protection from the acidic environment of the vagina.)

Which classes are appropriate for a 34-week pregnant patient and her partner who report being nervous about the upcoming delivery and baby care? A) Newborn care B) Prenatal exercise C) Early prenatal care D) Preconception care E) Childbirth preparation

A) Newborn care (A newborn care class should be recommended because it can teach expecting parents how to care for the infant. Adequate preparation and education can quell anxiety and fear.) E) Childbirth preparation (Childbirth preparation class should be recommended because adequate education regarding preparation for birth can help decrease the anxiety of the couple.)

Which drugs would the nurse anticipate administering for tocolysis during preterm labor? A) Nifedipine B) Indomethacin C) Magnesium sulfate D) Labetalol E) Betamethasone

A) Nifedipine (Nifedipine is given in large, frequent doses to blunt or stop uterine contractions to allow for administration of corticosteroids for fetal lung maturity prior to delivery.) B) Indomethacin (Indomethacin is utilized to blunt or stop uterine contractions during preterm labor. It is a short-term solution to allow for the administration of corticosteroids.)

The nurse is providing care for a patient in labor, and the health care provider has just stated the patient's need for a forceps-assisted delivery. Which actions would the nurse's preparation include? A) Obtaining a urinary catheter B) Establishing intravenous (IV) access C) Performing a head-to-toe assessment D) Educating the patient about the risk for lacerations E) Preparing the forceps using aseptic technique F) Monitoring the fetal heart rate for signs of distress

A) Obtaining a urinary catheter (The patient's bladder must be empty during an operative delivery. Use of an intermittent catheter is expected.) B) Establishing intravenous (IV) access (If possible, IV access should be established for the patient who undergoes an operative vaginal delivery. This can provide a route for additional pain medication. It is also necessary if the forceps delivery fails and cesarean delivery is necessary. Delivery should not be delayed to obtain IV access.) E) Preparing the forceps using aseptic technique (When an operative vaginal delivery is indicated, the nurse is often responsible for assisting the health care provider by preparing necessary supplies.) F) Monitoring the fetal heart rate for signs of distress (The health care provider is occupied with the delivery, and the nurse is responsible for alerting the health care provider to signs of fetal distress during operative vaginal delivery.)

In the fourth stage of labor, the nurse monitors the hemodynamic stability of the postpartum patient through which assessment? A) Obtaining maternal vital signs every 15 minutes in the first postpartum hour B) Assessing the couplet for bonding C) Obtaining maternal vital signs every 30 minutes in the first postpartum hour D) Assessing the color, amount, and odor of urine after the first void

A) Obtaining maternal vital signs every 15 minutes in the first postpartum hour (The maternal pulse, blood pressure, and respiratory rate should be assessed and documented every 15 minutes in the fourth stage of labor.)

At which time would it be most effective for the nurse to educate a new mother about circumcision care? A) On the second postpartum day B) With discharge teaching on the day of discharge C) During admission while going over the consent forms D) Within the first 24 hours after birth

A) On the second postpartum day (Education is most effective during the taking-hold phase, which begins 24 hours after birth. During this time, the mother begins to shift her focus from self-care to her infant.)

Which statements accurately characterize how the female body prepares for conception? A) Oocytes remain in the ovary in the graafian follicle. B) The ovum is released on the fallopian tube surface. C) An ovum is released 14 days before the menstrual period begins. D) The graafian follicle prepares the uterine lining by producing estrogen and progesterone. E) The ovum matures as follicle-stimulating hormone (FSH) and luteinizing hormone (LH) are released.

A) Oocytes remain in the ovary in the graafian follicle. (Each mature oocyte is contained in a sac within the ovary called the graafian follicle.) C) An ovum is released 14 days before the menstrual period begins. (Ovulation, or the release of the ovum, does occur about 14 days before a woman's next menstrual period begins.) D) The graafian follicle prepares the uterine lining by producing estrogen and progesterone. (The graafian follicle does produce estrogen and progesterone to prepare the endometrium (uterine lining) for a possible pregnancy.)

Which statements describe how the uterus, ovaries, and fallopian tubes function together to assist in conception? A) Ovaries mature the ovum. B) The uterus stores glycogen. C) Fallopian tubes transport the ovum. D) The uterus supports pelvic structures. E) The ovaries release follicle-stimulating hormone (FSH) and luteinizing hormone (LH).

A) Ovaries mature the ovum. (The ovaries develop an ovum to maturity during each reproductive cycle to assist in conception.) B) The uterus stores glycogen. (The uterus prepares for conception during each reproductive cycle by storing large quantities of glycogen, proteins, lipids, and minerals in the endometrium.) C) Fallopian tubes transport the ovum. (The fallopian tubes transport the ovum during each reproductive cycle to assist in conception.)

Which statements regarding prostaglandins (PGs) are accurate? A) PGs are produced by most organs of the body. B) PGs have an effect on ovulation. C) PGs influence production of estrogen. D) PGs are produced only in the uterus during pregnancy. E) PGs are produced by the graafian follicle.

A) PGs are produced by most organs of the body. (PGs are produced by most organs of the body, including the uterus.) B) PGs have an effect on ovulation. (There is indirect evidence that PGs have an effect on ovulation and fertility.) C) PGs influence production of estrogen. (After ovulation, PGs influence production of estrogen and progesterone by the corpus luteum.)

Which patient cues would be cause for concern when interviewing a patient regarding her intrauterine device that was placed 3 months ago? A) Patient reports her partner tested positive for chlamydia. B) A patient reports an unexplained fever and chills. C) The patient can feel the strings in her vagina with her fingers. D) The patient reports pelvic pain. E) The patient reports abnormal vaginal discharge.

A) Patient reports her partner tested positive for chlamydia. (An exposure to infection is a cause for concern, and the patient requires treatment to prevent infection.) B) A patient reports an unexplained fever and chills. (This could indicate infection and should be investigated.) D) The patient reports pelvic pain. (New-onset pelvic pain may be a sign of intrauterine infection secondary to the intrauterine device and should be investigated.) E) The patient reports abnormal vaginal discharge. (Abnormal vaginal discharge can be a sign of an infection related to an intrauterine device and should be investigated.)

Which auxiliary structures play a role in the elimination of fetal waste? A) Placenta B) Umbilical vein C) Amniotic fluid D) Fetal membranes E) Umbilical arteries

A) PlacentaThe placenta plays a key role in transferring carbon dioxide, urea, uric acid, and bilirubin from the fetus to the mother for disposal. E) Umbilical arteriesThe umbilical arteries carry blood high in carbon dioxide and other waste away from the fetus toward the placenta, where these waste products are removed through the maternal circulation.

While awaiting the delivery of the placenta in the third stage of labor, the nurse observes a large gush of blood and an increase in the length of the umbilical cord. Which event would the nurse recognize has probably occurred? A) Placental separation from the uterine wall B) Placental abruption C) Umbilical cord detachment from the placenta D) Postpartum hemorrhage as a result of retained placenta

A) Placental separation from the uterine wall (A large gush of blood and an increase in the length of the umbilical cord are signs of placental separation from the uterine wall.)

A child is born with a genetic disorder and is found to have 46 chromosomes. Which type of disorder could have caused the genetic disorder? A) Polygenic Disorder B) Single Gene Disorder C) Chromosomal Disorder D) Autosomal Recessive Disorder E) Autosomal Dominant Disorder

A) Polygenic Disorder (Polygenic disorders are characterized by changes in multiple genes, but the number of chromosomes remains the normal 46.) B) Single Gene Disorder (Single disorders are characterized by changes in one gene, but the number of chromosomes remains the normal 46.) D) Autosomal Recessive Disorder (Autosomal recessive disorders are characterized by changes in single genes, but the number of chromosomes remains the normal 46.) E) Autosomal Dominant Disorder (Autosomal dominant disorders are characterized by changes in single genes, but the number of chromosomes remains the normal 46.)

A G1/P0 gestational diabetic mother is undergoing induction of labor. She is in her 39th week of gestation, and she has been diagnosed with polyhydramnios. The nurse recognizes which patient cue as a risk factor for umbilical cord prolapse? A) Polyhydramnios B) Being a primigravida C) Gestational diabetes D) Term gestation

A) Polyhydramnios (An excessive amount of amniotic fluid, as with polyhydramnios, increases the risk for umbilical cord prolapse in labor.)

The nurse assessing for hypoglycemia in a neonate would monitor for which symptoms? A) Poor feeding B) Flexion of the arms and legs C) Hypothermia D) Tremors E) Bluish hands and feet F) Rooting

A) Poor feeding (Poor feeding and lack of interest in feeding are signs of hypoglycemia in the neonate and are usually related to lethargy caused by lack of glucose.) C) Hypothermia (Hypothermia is a symptom of hypoglycemia.) D) Tremors (Tremors may be observed in a newborn that has low blood glucose.)

Which underlying conditions are associated with amenorrhea? A) Pregnancy B) Endometriosis C) Tubal ligation following childbirth D) Cocaine use E) Stress

A) Pregnancy (Amenorrhea is a probable sign of pregnancy. Pregnancy should be ruled out if a woman presents with amenorrhea.) D) Cocaine use (The use of cocaine and other drugs such as marijuana and opiates is associated with amenorrhea.) E) Stress (Stress is associated with amenorrhea. The nurse should counsel the woman on ways to avoid or cope with stress.)

Which would the nurse assess at a routine prenatal visit for a patient that has gestational diabetes? A) Presence of bacteria, ketones, protein, and glucose in urine B) Fetal movement by palpation and patient report C) Serum calcium levels D) Blood pressure E) Fundal height

A) Presence of bacteria, ketones, protein, and glucose in urine (Women with diabetes are at increased risk for urinary tract infection. Additionally, the presence of ketones, glucose, and protein may indicate worsening diabetes or preeclampsia.) B) Fetal movement by palpation and patient report (Fetal movement is a reliable indicator of well-being. A patient that reports decreased fetal movement requires further evaluation.) D) Blood pressure (A patient with diabetes is at increased risk for preeclampsia, so the blood pressure should be assessed at each visit.) E) Fundal height (An elevated fundal height may indicate macrosomia or hydramnios and requires further diagnostic evaluation.)

A 36-week gestation patient presents with membranes grossly ruptured and is not contracting. Which diagnosis does the nurse anticipate? A) Preterm premature rupture of membranes (PPROM) B) Preterm labor (PTL) C) Premature rupture of membranes (PROM) D) AROM

A) Preterm premature rupture of membranes (PPROM) (PPROM is PROM that occurs before 36 weeks of gestation. Contractions may or may not be present. PPROM is often associated with PTL, with the greatest risks from preterm birth occurring before completing 34 weeks of gestation (ACOG, 2016).)

Which conditions are possible causes of dysfunctional labor? A) Psychological dysfunction and fear B) Absence of a void in 6 hours C) Rapid descent of the fetal head and small parts D) An abnormally shaped maternal pelvis E) Meconium-stained amniotic fluid F) Use of oxytocin to augment labor

A) Psychological dysfunction and fear (Maternal psychological problems can lead to a cascade of physiologic responses that can prolong labor.) B) Absence of a void in 6 hours (A full bladder is a possible cause because it causes maternal soft tissue obstruction.) D) An abnormally shaped maternal pelvis (An abnormally shaped maternal pelvis can cause dysfunctional contractions and slow labor progress.)

Which nursing action is appropriate for a menopausal woman who tells the nurse that she finds intercourse uncomfortable? A) Recommend the use of water-based lubricants before intercourse. B) Recommend abstinence from intercourse during menopause. C) Recommend the use of a condom during intercourse. D) Recommend an increase in fluid intake.

A) Recommend the use of water-based lubricants before intercourse. (During perimenopause, a woman may complain of vaginal dryness. This dryness may make intercourse uncomfortable; therefore the use of a water-based lubricant would lubricate the vagina without harsh chemicals.)

When administering nifedipine for tocolysis, it is important for the nurse to monitor for which serious side effect? A) Reflex tachycardia B) Hyperglycemia C) Change in fundal height D) Bradypnea

A) Reflex tachycardia (Reflex tachycardia (maternal pulse greater than 120) can occur with large doses of nifedipine.)

After assessment of an actively laboring woman, the nurse finds a fetal heart rate (FHR) of 180 beats/min with contractions occurring less than 2 minutes apart and lasting more than 90 seconds. Which action would the nurse take next? A) Reposition the patient to her side and alert the health care provider. B) Nothing, these are normal findings. C) Prepare for emergent cesarean delivery. D) Reposition the patient only.

A) Reposition the patient to her side and alert the health care provider. (The patient is experiencing tachysystole, fetal tachycardia, and incomplete uterine relaxation, which can lead to fetal compromise. Lateral positioning allows for optimal placental perfusion. The health care provider will need to evaluate this patient at the bedside.)

A nurse caring for a patient immediately postpartum after a precipitate labor would monitor the patient for which possible postpartum complication related to her precipitate labor? A) Retained placenta B) Infection C) Low Apgar scores D) Postpartum depression

A) Retained placenta (A patient who experiences precipitate labor is at increased risk for retained placenta secondary to a rapid delivery.)

The nurse understands that which hormonal changes are thought to encourage the onset of labor? A) Rising estrogen counteracts the relaxing effect of progesterone on the uterus. B) Follicle-stimulating hormone is secreted in large quantities by the anterior pituitary. C) Prostaglandins are secreted by the fetal membranes preparing the uterus. D) The fetus secretes cortisol, which may act as an additional uterine stimulant. E) Thyroid-stimulating hormone is enhanced and helps make the uterus contractile. F) Oxytocin is secreted by the mother, which causes uterine contractions.

A) Rising estrogen counteracts the relaxing effect of progesterone on the uterus. (Rising estrogen levels counteract the action of progesterone. When estrogen levels rise above progesterone levels, the relaxant effects of progesterone on the uterine muscle are reduced. This enables uterine contractions to begin.) C) Prostaglandins are secreted by the fetal membranes preparing the uterus. (Prostaglandins are secreted from the lower area of the fetal membranes (forebag) during labor, which prepares the uterus for oxytocin stimulation.) D) The fetus secretes cortisol, which may act as an additional uterine stimulant. (The fetal membranes release prostaglandin in high concentrations during labor. In addition to fetal oxytocin secretion, large quantities of cortisol are secreted by the fetal adrenal gland, possibly acting as a uterine stimulant.) F) Oxytocin is secreted by the mother, which causes uterine contractions. (The natural increase in secretion of oxytocin from the mother and the increase in oxytocin availability resulting from the compression of the fetal head on the cervix furthers and maintains labor once it has begun. Oxytocin receptors in the uterus increase markedly during labor and peak at delivery, allowing oxytocin to have its effect on the uterus.)

Wound dehiscence is a sign of a wound infection and is characterized by which finding? A) Separation of the wound site B) Purulent drainage at the wound site C) Bleeding at the wound site D) Edema at the wound site

A) Separation of the wound site (Wound dehiscence is also known as wound separation and can indicate a wound infection. Wound dehiscence should be reported immediately to a health care provider.)

Which sign would lead the nurse to suspect ectopic pregnancy in a patient with a missed period? A) Severe, localized abdominal pain in the left lower abdominal quadrant B) Vaginal bleeding after intercourse C) Nausea and vomiting D) Painless, bright-red vaginal bleeding

A) Severe, localized abdominal pain in the left lower abdominal quadrant (Localized and severe pain in the lower abdominal quadrants may indicate ectopic pregnancy in the proximal end of the fallopian tube.)

Which type of anemia is an inherited anemia? A) Sickle cell anemia B) Aplastic anemia C) Iron deficiency anemia D) Pernicious anemia

A) Sickle cell anemia (Sickle cell anemia is an autosomal recessive disorder.)

Which infant actions are common cues indicating a readiness to feed? A) Sucking B) Rooting C) Mouthing D) Crying E) Hand-to-mouth movements F) Vocalizations

A) Sucking (Sucking is a common feeding cue.) B) Rooting (The rooting reflex is a common feeding cue characterized by the newborn turning and rooting with the mouth when the cheek or lip is touched.) C) Mouthing (Mouthing is a common feeding cue characterized by the newborn sucking when the mouth is stimulated.) E) Hand-to-mouth movements (Hand-to-mouth movements are a common feeding cue characterized by bringing hand(s) to the mouth.)

Which are the advantages of external electronic fetal monitoring (EFM)? A) Suitable for most patients in labor B) Visualization of heart rate pattern C) Allows measurement of contraction intensity D) Uninterrupted by fetal or patient movement E) Allows mobility if connected to mobile unit

A) Suitable for most patients in labor (External EFM is a noninvasive method for continuously monitoring fetal heart rate and uterine contractions. It is safe to use in high-risk pregnancies.) B) Visualization of heart rate pattern (External EFM provides visualization of the fetal heart rate pattern in response to uterine contractions.) E) Allows mobility if connected to mobile unit (Mobile external EFM units allow patients to remain mobile, as the unit may be wheeled during ambulation.)

Which factors are likely to support breastfeeding as the family's feeding method choice? A) Support from family B) Formula access C) Cultural practices D) Return to work E) Health care provider knowledge

A) Support from family (Support and encouragement from family and friends is a major influence on feeding method choice and has been shown to increase breastfeeding choice.) C) Cultural practices (Cultural practices are highly influential on feeding method choice. Duration of breastfeeding is also highly influenced by culture.) E) Health care provider knowledge (Health care provider breastfeeding knowledge is highly influential on the choice to breastfeed because the health care provider may assist with establishing breastfeeding, troubleshooting common challenges, and sharing advantages of breastfeeding.)

Which observations suggest that a woman is the latent phase of the second stage of labor? A) The fetus is at 0 station. B) There is an irregular and inconsistent urge to bear down with contractions. C) The woman exhibits loud, songlike vocalizations with every contraction. D) The fetus is at +3 station. E) The woman sleeps between contractions. F) The woman is pushing involuntarily.

A) The fetus is at 0 station. (Ferguson reflex is not usually present until the fetus begins to descend deeper into the pelvis. Latent labor allows for passive descent with the primary force being contractions.) B) There is an irregular and inconsistent urge to bear down with contractions. (The Ferguson reflex is not usually present until the fetus begins to descend deeper into the pelvis. Latent labor allows for passive descent with the primary force being contractions.) E) The woman sleeps between contractions. (In the latent phase of the second stage of labor, maternal behaviors may be relaxed. She may be very quiet as she rests in this phase. The woman may sleep or rest in between contractions.)

Which statement regarding pain experienced during labor is accurate? A) The predominant pain of the first stage of labor is the visceral pain located in the lower portion of the abdomen. B) Referred pain is the extreme discomfort between contractions. C) The somatic pain of the second stage of labor is more generalized and related to fatigue. D) Pain during the third stage is a somewhat milder version of the second stage.

A) The predominant pain of the first stage of labor is the visceral pain located in the lower portion of the abdomen. (This pain comes from cervical changes, distention of the lower uterine segment, and uterine ischemia.)

Which statements regarding the secretion of human chorionic gonadotropin (hCG) are accurate? A) The zygote secretes hCG. B) A high level of hCG is a pregnancy indicator. C) Pregnant women have high levels of hCG and progesterone. D) Ovulating women have higher amounts of hCG in their system. E) Increased hCG production causes decreased estrogen secretion.

A) The zygote secretes hCG. (The zygote secretes hCG to maintain the decidua.) B) A high level of hCG is a pregnancy indicator. (The zygote secretes hCG to signal that a pregnancy has begun; therefore a high level of hCG is an indicator of pregnancy.) C) Pregnant women have high levels of hCG and progesterone. (Implantation and survival of the conceptus are critically dependent on a continuing supply of estrogen and progesterone to maintain the decidua in the secretory phase. With continued hCG production by the conceptus, the corpus luteum continues to secrete estrogen and progesterone rather than regressing.)

How is effacement measured? A) Through digital palpation by a trained professional B) Using transvaginal ultrasound C) By estimating the stage of labor D) Though palpation of cervical dilation E) Through Leopold maneuvers

A) Through digital palpation by a trained professional (Digital palpation and transvaginal ultrasound can estimate cervical effacement.) B) Using transvaginal ultrasound (Digital palpation and transvaginal ultrasound can estimate cervical effacement.)

Which phrases describe the goals of perinatal education classes? A) To decrease anxiety related to childbirth B) To teach parents how to perform cardiopulmonary resuscitation (CPR) for their children C) To provide knowledge related to the labor process and birth D) To increase knowledge of comfort relief measures during labor E) To teach the expectant families how to parent and discipline their children

A) To decrease anxiety related to childbirthDecreasing anxiety related to childbirth is a goal of perinatal education classes. C) To provide knowledge related to the labor process and birthProviding knowledge related to the labor process and birth is a goal of perinatal education classes. D) To increase knowledge of comfort relief measures during laborIncreasing knowledge of comfort relief measures during labor is a goal of perinatal education classes.

Which passenger position would interfere with a safe vaginal birth? A) Transverse fetal lie B) Flexed fetal head C) Cephalic presentation D) Longitudinal fetal lie

A) Transverse fetal lie (The fetal lie is the relationship of the long axis of the baby to the long axis of the mother. With a transverse fetal lie, there is a significant risk for fetal distress, trauma, and maternal trauma if vaginal delivery occurs.)

Which contraception methods would be appropriate for a woman who is intolerant to hormonal therapy? A) Tubal ligation B) Emergency contraception (EC) C) Contraceptive vaginal ring D) Diaphragm with spermicide E) Copper intrauterine device (IUD)

A) Tubal ligation (Tubal ligation is a surgical procedure that uses no hormones, so the procedure is an appropriate method of contraception for a woman who is intolerant to hormonal therapy.) D) Diaphragm with spermicide (A diaphragm does not contain hormones, nor does the spermicide used with it, and the combination of the two would be appropriate for a woman who is intolerant to hormonal therapy.) E) Copper intrauterine device (IUD) (A copper IUD does not contain any hormones and is appropriate for a woman who is intolerant to hormonal therapy.)

Which two factors trigger cardiovascular adaptation after birth? A) Umbilical cord clamping B) Tactile stimulation C) Chest compression D) Initiation of respiration E) Chemical factors

A) Umbilical cord clamping (Umbilical cord clamping functionally closes the umbilical arteries, umbilical vein, and ductus venosus.) D) Initiation of respiration (Initiation of respiration inflates the lungs, decreases pulmonary vascular resistance, and stimulates increased pulmonary blood flow, leading to closure of the ductus arteriosus and foramen ovale.)

Which characteristic describes sleep disturbances associated with postpartum depression? A) Unrestful sleep B) Caused by the baby's presence C) Excessive sleeping D) Attributed to the need for nighttime feedings

A) Unrestful sleep (There is a different quality to sleep problems in women with postpartum depression. It is usual for new mothers to have their sleep interrupted by a crying baby, but women with postpartum depression report that they cannot go to sleep even when the baby is settled and goes to sleep.)

When witnessing informed consent for cesarean delivery, which risks to the patient is the nurse acknowledging the patient understands? A) Urinary tract infection or trauma to the bladder B) Cervical laceration or trauma C) Increased risk for blood clots, which can be in the legs or can travel to the heart or lungs D) Injury to the newborn including lacerations, bruising, or fractures E) The need for cesarean birth with every delivery after this procedure regardless of the incision type F) The risk that the baby's lungs might be compromised or need extra support at birth

A) Urinary tract infection or trauma to the bladder (An indwelling catheter increases the risk for urinary tract infection, and there is risk for trauma to the bladder during the procedure.) C) Increased risk for blood clots, which can be in the legs or can travel to the heart or lungs (Major surgery such as cesarean section increases a patient's risk for thrombophlebitis and thromboembolism.) D) Injury to the newborn including lacerations, bruising, or fractures (Cesarean delivery can be traumatic for the newborn, especially if the delivery is complicated by malpresentation, macrosomia, or other factors.) F) The risk that the baby's lungs might be compromised or need extra support at birth (There is risk for inadvertent fetal lung immaturity or transient tachypnea of the newborn with cesarean delivery.)

A woman, 1 day postpartum, is being carefully monitored after a significant postpartum hemorrhage (PPH). Which finding would the nurse report to the health care provider? A) Urine output of 160 mL for the past 8 hours B) Weight loss of 2 lb since delivery C) Pulse rate of 68 beats/min D) Fundus firm at the umbilicus

A) Urine output of 160 mL for the past 8 hours (Oliguria (urine output <30 mL/h) should be reported, as this is a late sign of hypovolemic shock.)

The antepartum nurse would provide the pregnant woman education on which signs and signals to report immediately to her health care provider? A) Vaginal bleeding B) Rupture of membranes C) Heartburn accompanied by a severe headache D) Fetal movement E) White, acidic discharge

A) Vaginal bleeding (Vaginal bleeding is considered a potential complication of pregnancy and may indicate spontaneous abortion, ectopic pregnancy, placenta previa, placental abruption, or uterine rupture. The pregnant woman should notify her health care provider if she has any vaginal bleeding throughout the pregnancy.) B) Rupture of membranes (Rupture of membranes, before the 37th week, is considered a potential complication and may indicate preterm premature rupture of membrane (PPROM). The pregnant woman should notify her health care provider for any sudden discharge of fluid from the vagina.) C) Heartburn accompanied by a severe headache (Heartburn (epigastric pain) coupled with a severe headache is considered a potential complication of pregnancy and may indicate hypertensive conditions, preeclampsia, or placental abruption. A pregnant woman should notify her health care provider if she experiences heartburn accompanied by a severe headache.)

Which factor explains the cause of hot flashes during menopause? A) Vasomotor instability B) Increased estrogen production C) Progesterone secretion D) Changes to the hypothalamus

A) Vasomotor instability (Perimenopause can cause vasomotor instability (which results in symptoms such as hot flashes) and is correlated with serum follicle-stimulating hormone levels.)

Which clinical finding is considered a positive sign of pregnancy? A) Visualization of the fetus by ultrasound B) Positive serum pregnancy test C) Breast enlargement D) Braxton Hicks contractions

A) Visualization of the fetus by ultrasound (The visualization of a fetus via an ultrasound transducer is considered a positive sign of pregnancy because there is no alternative explanation to the finding.)

Which nonmedical therapy could be recommended for a woman experiencing infertility? A) Weight loss of 15 lb B) Assisted reproductive technology (ART) C) Infertility support group D) Vegan diet

A) Weight loss of 15 lb (Women who are overweight or obese have a reduced chance of pregnancy and a significantly greater risk for spontaneous abortion following infertility treatment compared with women of normal weight. Even modest weight loss (5% to 10%) can be sufficient to increase their chances of achieving a successful pregnancy.)

A patient with preeclampsia has an emergency delivery at 32 weeks gestation. Which findings would the nurse anticipate in this premature neonate? A) Yellow skin B) Weight of 2000 g C) Axillary temperature of 97.7°F D) Heart rate of 120 beats/min E) Poor respiratory effort

A) Yellow skin (Yellow skin indicates hyperbilirubinemia, or jaundice, which is a condition in which too much bilirubin is in the blood. This can result from a mother with preeclampsia.) B) Weight of 2000 g (A full-term baby normally weighs at least 2500 g. A weight of 2000 g would be expected for a preterm baby born to a mother with preeclampsia.) E) Poor respiratory effort (Poor respiratory rate is associated with premature neonates because the lungs may be underdeveloped.)

Match the fetal surveillance method to its corresponding advantage. Accurate measurement of fetal heart rate Accurate measurement of uterine contractions, including intensity Noninvasive and suitable for most patients in labor Intrauterine pressure catheter; Fetal scalp electrode; External electronic fetal monitoring

Accurate measurement of fetal heart rate: Fetal scalp electrode Accurate measurement of uterine contractions, including intensity: Intrauterine pressure catheter Noninvasive and suitable for most patients in labor: External electronic fetal monitoring

Which nursing response is appropriate for the mother of a 12-year-old who is concerned that her daughter has not started her period? A) "Your daughter may be experiencing primary amenorrhea. Please make an appointment with the health care provider as soon as possible." B) "Although the average age of menses is 12 years, some girls may not begin menses until age 16 years." C) "Your daughter may be experiencing dysmenorrhea and should be evaluated by the health care provider as soon as possible." D) "Your daughter should start her period at the same age that you started your period. What age did you begin menstruating?"

B) "Although the average age of menses is 12 years, some girls may not begin menses until age 16 years." (The average age of menses is age 12 years. If she does not begin menstruating by age 16 years, further evaluation should be conducted to rule out the presence of anatomic abnormalities or diseases.)

A woman had an emergency cesarean delivery for a prolapsed cord. Both the mother and infant are stable, and the nurse is caring for the mother and infant couplet. Which statement by the mother indicates she is in the taking-in phase? A) "Do you think the baby is breathing OK? His breathing seems very fast, and then it slows down. Is this normal?" B) "I can't wait for my sister to come with food and take pictures of the baby while I rest." C) "Not feeling movement in my stomach seems so different now. I didn't know how much the baby moved when I was pregnant." D) "The baby is so small that I worry that I will hurt him when I change his diaper. He moves so much!"

B) "I can't wait for my sister to come with food and take pictures of the baby while I rest." (The mother's statement that she can't wait for her sister to bring food and give her a chance to rest reflects the taking-in phase, in which the mother reflects on herself and her recovery needs.)

Which statement by a woman with multiple sexual partners after a tubal ligation would indicate that additional teaching regarding preventive screening is necessary? A) "I will come yearly for a Papanicolaou (Pap) test." B) "I don't have to worry about using condoms for sexually transmitted infections (STIs)." C) "I am continuing to do monthly vulvar self-examinations." D) "I don't have to get a colonoscopy."

B) "I don't have to worry about using condoms for sexually transmitted infections (STIs)." (A tubal ligation prevents pregnancy from occurring but does not protect a woman with multiple sexual partners from STIs.)

At a prenatal checkup, a woman at 26 weeks gestation expresses concern that her baby is in breech position as revealed by a sonogram. Which response by the nurse explains fetal position changes at this gestational age? A) "Don't worry; babies typically flip after 30 weeks." B) "I understand your concern, but babies usually do not flip until about week 28." C) "We need to plan for a cesarean delivery, as babies assume their birthing position now." D) "The baby's head is bigger than the feet but not heavy enough to cause him to turn yet."

B) "I understand your concern, but babies usually do not flip until about week 28." (The fetus floats freely but begins to be positioned head down between weeks 25 and 28; therefore the baby may still flip.)

Which nursing information is appropriate when teaching a postpartum woman about how to increase perineal muscle tone? A) "Ambulate early and frequently." B) "Perform Kegel exercises 30 times per day." C) "Perform pelvic lifts several times per day." D) "Tighten and relax the abdominal muscles several times per day."

B) "Perform Kegel exercises 30 times per day." (Performing Kegel exercises 30 times per day will increase perineal muscle tone.)

A 47-year-old woman presents at a clinic complaining of irregular periods and states, "I am too young for menopause." Which is the best response from the nurse? A) "Yes, menopause occurs only after age 50." B) "Some women go through menopause earlier than others." C) "The health care provider will do a test to determine the cause." D) "Yes, menopause occurs only after age 60."

B) "Some women go through menopause earlier than others." (The average age of natural menopause is 51.4 years, and perimenopause can last for 4 years. Therefore this woman could be in perimenopause.)

Which educational information related to activity would the nurse provide to a postpartum woman who delivered by cesarean birth? A) "Housework may be resumed within 2 weeks." B) "Stair-climbing should be limited for the first 2 weeks." C) "Leg muscles should be used to lift rather than the abdominal muscles." D) "Walking and swimming are encouraged to prevent blood clot formation." E) "Abdominal exercises should be done several times per day after discharge to increase muscle tone."

B) "Stair-climbing should be limited for the first 2 weeks." (Stair-climbing should be limited for the first 2 weeks postpartum to allow healing of the abdominal incision.) C) "Leg muscles should be used to lift rather than the abdominal muscles." (The leg muscles should be used for lifting the first few weeks postpartum to allow for healing of the abdominal incision.)

In which situation is there an increased likelihood for prolonged labor? A) The woman is a teen mother. B) The woman is nulliparous. C) The woman has been diagnosed with an incompetent cervix. D) The woman has a history of postpartum hemorrhage.

B) The woman is nulliparous. (A woman who has never before given birth vaginally is at increased risk for prolonged labor.)

Which explanation of cesarean delivery incisions would a nurse provide to a patient? A) "There are a total of three incisions that will be made in your body by the surgeon." B) "The surgeon may make a vertical incision or a transverse incision on your skin." C) "You will not be able to the see the incision once it is fully healed if the surgeon makes a Pfannenstiel incision on your skin." D) "If you have a transverse incision on your uterus, you can never have a vaginal birth."

B) "The surgeon may make a vertical incision or a transverse incision on your skin." (A transverse (Pfannenstiel) incision or vertical incision may be made on the skin at the discretion of the surgeon based on various considerations.)

A patient with a 2-year-old child with cystic fibrosis declines genetic counseling during her antepartum visit, stating that the birth defect has "already happened," and therefore her future children will not be affected. Which is the appropriate nursing education for this patient? A) "The risk for a child developing cystic fibrosis increases with maternal age." B) "This infant has an equal risk for developing cystic fibrosis as your first child." C) "The risk for your infant developing cystic fibrosis is less than your first child, but it is still present." D) "There is no longer a risk for your infant developing cystic fibrosis, but genetic counseling can help detect other abnormalities."

B) "This infant has an equal risk for developing cystic fibrosis as your first child." (Having one child with a birth defect does not change the risk for subsequent children developing the same defect. This response clarifies the patient's misconception about genetic factors of cystic fibrosis and is appropriate for the patient.)

Which nursing recommendation is appropriate for a woman experiencing pain associated with endometriosis? A) "Take oral oxycodone/acetaminophen for pain every 4 hours around-the-clock." B) "You may take regular strength ibuprofen for pain every 8 hours." C) "The use of laxatives will help reduce the pain." D) "You should not take any medications when you have pain associated with endometriosis because they may cause addiction."

B) "You may take regular strength ibuprofen for pain every 8 hours." (Use of nonsteroidal antiinflammatory drugs (NSAIDs), such as ibuprofen, is an appropriate recommendation for a woman experiencing pain associated with endometriosis.)

Which nursing response is appropriate for a 20-week gestation woman who is concerned about gaining 12 lb (5.4 kg) in her pregnancy? A) "I will tell the health care provider that you are worried. The health care provider will tell you what to do." B) "Your weight gain is about average for this point in your pregnancy. What concerns you about it?" C) "You really have gained a lot! I will consult with the nutritionist for you." D) "A lot of your weight gain is probably fluid. I suggest you decrease your fluid intake and increase your salt intake."

B) "Your weight gain is about average for this point in your pregnancy. What concerns you about it?" (Average weight gain at 20 weeks is 9 to 12 lb (4 to 5.4 kg). Finding out the pregnant woman's concerns is important because she is within normal limits for weight gain at this gestation.)

The nurse is instructing a childbirth class on abdominal breathing and tells a patient that her baseline respiratory rate is 22 breaths/min. What should the patient's rate be while performing slow-paced breathing techniques? A) 9 breaths/min B) 11 breaths/min C) 15 breaths/min D) 20 breaths/min

B) 11 breaths/min (The ideal rate for slow-paced breathing is half the normal breathing rate. Because her baseline respiratory rate is 22 breaths/min, the ideal breathing rate for slow-paced breathing is 11 breaths/min.)

Which woman has an increased risk for developing preeclampsia? A) A 25-year-old woman with liver disease B) A 26-year-old woman with a body mass index (BMI) of 32 C) A 34-year-old woman who has breast cancer D) A 30-year-old woman with a family history of diabetes

B) A 26-year-old woman with a body mass index (BMI) of 32 (Overweight patients have an increased risk for developing preeclampsia.)

A nurse is caring for a patient in labor who is receiving oxytocin via intravenous infusion to stimulate uterine contractions. Which assessment finding would indicate to the nurse that the infusion needs to be discontinued? A) Three contractions occurring within a 10-minute period B) A fetal heart rate of 90 beats/min C) A resting uterine tone of 15 mm Hg via the intrauterine pressure catheter D) Early decelerations

B) A fetal heart rate of 90 beats/min (A normal fetal heart rate is 110 to 160 beats/min. Bradycardia and/or late or variable decelerations indicate fetal distress and the need to discontinue the oxytocin infusion.)

What would the nurse expect when caring for a full-term primigravida who presents for a routine office visit and is not in labor? A) A fundal height that measures 30 cm B) A fetus that is engaged in the maternal pelvis C) A cervix that is already 3 cm dilated D) Fetal station of −3

B) A fetus that is engaged in the maternal pelvis (Fetal engagement often occurs before the beginning of labor in primigravidas, where with multigravidas the fetus often does not engage until labor begins.)

An Rh-negative woman needs an amniocentesis. The nurse understands which statement to be true based upon the information provided? A) The amniocentesis will determine if the mother needs a blood transfusion. B) A shot of Rho(D) immune globulin will be required for this procedure. C) A Coombs test should be performed immediately after the procedure. D) The fetus is likely Rh-positive, so delivery should be expedited.

B) A shot of Rho(D) immune globulin will be required for this procedure. (As a result of the potential for maternal fetal hemorrhage, Rho(D) immune globulin should be given after an amniocentesis when a woman is Rh-negative.)

A nurse on the postpartum unit is caring for several postpartum women. Which woman would the nurse recognize as having the greatest risk for developing a postpartum infection? A) A woman who experienced a precipitous labor less than 3 hours in duration B) A woman who has type 1 diabetes and delivered a large-for-gestational-age infant via cesarean section C) A woman who had a boggy uterus that was not well contracted after delivery D) A woman who has a first-degree laceration of the perineum

B) A woman who has type 1 diabetes and delivered a large-for-gestational-age infant via cesarean section (Both type 1 diabetes and a cesarean section increase a woman's risk for developing a postpartum infection such as endometritis.)

In which areas of the body do the events of the menstrual cycle occur concurrently? A) The endometrium, hypothalamus, and ovaries B) The endometrium, hypothalamus, pituitary gland, and ovaries C) The endometrium, pituitary gland, and ovaries D) The endometrium, hypothalamus, gonads, and ovaries

B) The endometrium, hypothalamus, pituitary gland, and ovaries (The menstrual cycle is a complex event that occurs concurrently within the endometrium, hypothalamus, pituitary gland, and ovaries.)

Which methods can all women use to reduce risk in the preconception period? A) Increasing saturated fats in their diet B) Abstaining from alcohol C) Taking a folic acid supplement D) Learning about inherited conditions that can affect pregnancy E) Achieving a healthy weight F) Waiting to update vaccines until after pregnancy to prevent exposure to teratogens

B) Abstaining from alcohol (Abstaining from alcohol for at least 1 month before conceiving and throughout pregnancy can prevent complications from fetal alcohol exposure.) C) Taking a folic acid supplement (Folic acid deficiencies are linked to open neural tube defects. All women of childbearing age who may become pregnant should take a supplement.) D) Learning about inherited conditions that can affect pregnancy (Certain conditions can be passed on to offspring or can adversely affect a pregnancy.) E) Achieving a healthy weight (Being severely overweight or underweight has serious health consequences for the mother and fetus.)

During an emergency cesarean section, which interventions can help minimize maternal risk? A) Eliminating surgical instrument counts to expedite the procedure B) Administering antibiotics before skin incision C) Performing the Foley catheter insertion using clean technique to expedite the procedure D) Performing an interdisciplinary time-out before skin incision E) Having a prewarmed isolette available for the birth

B) Administering antibiotics before skin incision (Administering antibiotics prior to skin incision is a standard of care, and it helps to prevent postoperative infection.) D) Performing an interdisciplinary time-out before skin incision (An interdisciplinary time-out allows for proper patient identification, interdisciplinary collaboration, and identification of patient-specific surgical risks and allergies, and it helps minimize maternal risk.)

A postpartum patient who had a spinal block for a scheduled cesarean delivery complains of a headache. Which intervention would the nurse anticipate? A) Oral (PO) administration of promethazine B) Administration of blood patch by the anesthesia provider C) Having patient sit up in chair to help relieve headache D) Allowing the spinal headache to run its course

B) Administration of blood patch by the anesthesia provider (Administration of a blood patch by an anesthesia professional is a procedure in which the patient's blood is injected into the spinal space, creating a seal over the site where the dural puncture occurred.)

A woman is 10 cm dilated and feels occasional rectal pressure at the peak of a contraction, and the fetal station is 0. There is a reassuring fetal heart tracing. Which nursing action is appropriate at this time? A) Recheck the cervical dilation to ensure she is 10 cm. B) Allow the woman to labor down for passive descent. C) Encourage the woman to push with every contraction. D) Prepare for cesarean delivery for cephalopelvic disproportion.

B) Allow the woman to labor down for passive descent. (The latent phase may also be referred to as laboring down, delayed pushing, or passive descent. The woman should be encouraged to rest until she feels the strong urge to push with each contraction.)

A woman is at 36 weeks gestation when her toddler hits her in the abdomen with a Wiffle-ball bat. Which auxiliary component protects the fetus from this impact? A) Placenta B) Amniotic fluid C) Amnion membrane D) Chorion membrane

B) Amniotic fluid (Amniotic fluid is an auxiliary component composed of a protective liquid and contained by the amniotic sac; it provides cushioning from maternal abdominal impact.)

Which antihypertensive classifications are contraindicated in pregnancy? A) Beta blockers B) Angiotensin-converting enzyme (ACE) inhibitors C) Calcium channel blockers D) Vasodilators E) Angiotensin receptor blockers (ARBs)

B) Angiotensin-converting enzyme (ACE) inhibitors (ACE inhibitors have been linked to adverse fetal effects and are contraindicated in pregnancy.) E) Angiotensin receptor blockers (ARBs) (ARBs have been linked to fetal morbidity and are not approved for use in pregnancy.)

The nurse assesses the blood pressure of a pregnant patient with preeclampsia and notes that is elevated. Which antihypertensive classification would be avoided for this patient? A) Calcium channel blockers B) Angiotensin-converting enzyme (ACE) inhibitors C) Vasodilators D) Beta blockers

B) Angiotensin-converting enzyme (ACE) inhibitors (ACE inhibitors may cause renal damage to a developing fetus, and they are not used in pregnancy.)

Which care would the nurse provide for an infant with macrosomia? A) Administer intravenous (IV) glucose. B) Assess for birth trauma. C) Monitor thermoregulation. D) Educate families on proper handling. E) Prepare for mechanical ventilation.

B) Assess for birth trauma. (Macrosomia, or an infant who is large-sized, increases the likelihood of birth trauma such as clavicle fracture or brachial plexus injury; therefore the nurse would assess for such injuries.) C) Monitor thermoregulation. (Infants with macrosomia are at risk for thermal dysregulation due to large body surface area.) D) Educate families on proper handling. (The infant may be large but is also physiologically immature; therefore the nurse educates families on proper handling.)

Which qualities are considered normal when assessing fetal heart rate (FHR)? A) FHR variability of <5 beats/min B) Baseline FHRof 140 beats/min C) FHR variability of 20 beats/min D) Baseline FHR of 100 beats/min E) Baseline FHR of 170 beats/min

B) Baseline FHRof 140 beats/min (A baseline FHR of 110 to 160 beats/min is considered normal.) C) FHR variability of 20 beats/min (FHR variability of 6 to 25 beats/min is classified as moderate variability and is considered normal (reassuring).)

Which placental variations present a risk for fetal hemorrhage? A) Discoid B) Battledore C) Vasa previa D) Velamentous E) Decidua basalis

B) Battledore (The battledore placental variation occurs when the umbilical cord is inserted near the margin of the placenta, causing an increased risk for fetal hemorrhage.) C) Vasa previa (The vasa previa placental variation occurs when the cord vessels branch far out on the membranes. When membranes rupture, fetal umbilical vessels may be torn, and the fetus can hemorrhage.) D) Velamentous (The velamentous placental variation occurs when the cord vessels branch at the membranes and then enter the placenta; with this variation, there is the potential for the fetal vessel to rupture, causing fetal hemorrhage.)

Which nursing intervention is appropriate to help prevent thrombophlebitis in a postpartum woman who delivered by cesarean birth? A) Administer warfarin therapy by mouth. B) Encourage early and frequent ambulation. C) Administer intravenous (IV) furosemide to increase urinary output. D) Encourage increased fluid intake.

B) Encourage early and frequent ambulation. (Early and frequent ambulation helps prevent thrombophlebitis caused by venous stasis and hypercoagulation in postpartum women who delivered by cesarean birth, who have double the risk.)

A woman at 37 weeks gestation calls and reports, "My water broke and I have bloody show. I am changing my pad every 10 minutes." Which advice would the nurse provide in response? A) Engage in activity such as walking. B) Call her health care provider and go to the hospital or birthing center. C) Advise the woman that overreacting is not good for her or the baby and to call back when she is in "true labor." D) Remain home, and only come in to the hospital or birthing center if contractions are regular, frequent, and intense.

B) Call her health care provider and go to the hospital or birthing center. (Heavy bleeding and/or a gush of fluid from the vagina should be evaluated by a health care provider at the hospital or birthing center.)

Which description is true of congenital cytomegalovirus (CMV)? A) Can lead to severe maternal anemia B) Can lead to neonatal hearing loss C) Often causes severe flulike symptoms in the mother D) Causes weeping skin lesions on the neonate

B) Can lead to neonatal hearing lossCMV is a leading cause of neonatal hearing loss.

A nurse is concerned that her pregnant patient is at risk for dehydration due to acute morning sickness. Which intervention is most appropriate in order to anticipate a treatment plan? A) Is the patient allergic to promethazine? B) Can the patient tolerate oral food and fluids? C) Does the patient have leukocytes in her urine? D) Does the patient have a patent intravenous (IV) site?

B) Can the patient tolerate oral food and fluids? (The nurse should determine whether the patient is able to tolerate oral fluids to treat acute morning sickness in the pregnant patient. Patients who are able to tolerate oral fluids should start with small amounts of cold water or sports drinks and increase gradually until the dehydration subsides.)

Which nursing interventions are appropriate for an infant experiencing withdrawal? A) Swaddle with the legs extended. B) Cluster care to minimize handling. C) Reduce environmental stimuli. D) Offer feedings every 3 to 4 hours. E) Offer nonnutritive sucking.

B) Cluster care to minimize handling. (Clustered care to minimize handling is an appropriate intervention for an infant experiencing withdrawal; infants are easily startled and distressed.) C) Reduce environmental stimuli. (Reduction of environmental stimuli, such as lights or noise, is appropriate to avoid startling or distressing the infant experiencing withdrawal.) E) Offer nonnutritive sucking. (Nonnutritive sucking is appropriate because it is soothing for infants experiencing withdrawal.)

Which type of commercial formula requires dilution with equal parts water? A) Ready-to-feed B) Concentrate C) Powder D) Hydrolysate formula

B) Concentrate (Concentrate formula requires dilution with equal parts of formula and water.)

Which cause of spontaneous abortion is the most common? A) Infection B) Congenital anomalies C) Vaginal bleeding D) Anomalies of the maternal reproductive tract

B) Congenital anomalies (Severe congenital abnormalities that are incompatible with life are the most common causes of spontaneous abortions.)

A nurse is beginning to care for a patient in labor. The health care provider has prescribed an intravenous (IV) infusion of oxytocin. The nurse makes sure that which implementations occur before initiation of the infusion? A) Placing the patient on complete bed rest B) Continuous electronic fetal monitoring C) An IV infusion of antibiotics D) Making sure that a vial of terbutaline is immediately available E) Preparing an IV infusion pump F) Placing a code cart at the patient's bedside

B) Continuous electronic fetal monitoring (Continuous electronic fetal monitoring, either external or internal, is a standard of practice with the administration of IV oxytocin.) D) Making sure that a vial of terbutaline is immediately available (Terbutaline, administered either intravenously or subcutaneously, is a tocolytic that can be used to decrease or stop uterine contractions when uterine tachysystole occurs.) E) Preparing an IV infusion pump (IV oxytocin should only be administered via an IV pump. IV oxytocin during labor should never be administered without the use of a pump for titration.)

Which assessment findings in the laboring patient would suggest to the nurse that delivery is imminent? A) The woman can talk through contractions. B) Contractions are 1.5 to 2 minutes apart. C) Regular contractions are mild and 7 minutes apart. D) Mucous vaginal discharge is bloody. E) The woman is experiencing vomiting, nausea, and sweating. F) The patient complains of rectal pressure.

B) Contractions are 1.5 to 2 minutes apart. (This is common in active labor.) D) Mucous vaginal discharge is bloody. (Bloody show is a sign of labor progress and often appears in active labor.) E) The woman is experiencing vomiting, nausea, and sweating. (These are signs that a patient is in active labor.) F) The patient complains of rectal pressure. (This is a sign of fetal descent and often happens in active labor.)

A G2/P1 woman presents for evaluation. She is excited, can talk through regular contractions every 7 to 10 minutes, and states that her pain is "manageable." She does not desire epidural anesthesia in labor. The nurse suspects this patient is in which phase of labor? A) Transitional labor B) Early labor C) The second stage of labor D) Active labor

B) Early labor (In early labor a woman may present for evaluation as excited and able to talk and walk through most contractions, though they are still uncomfortable. She may be focused on herself and her baby. She may follow directions well and pain may be easily managed.

A patient in labor experiences minimal relief after nonpharmacologic interventions, and she is not coping well in labor. Which action would the nurse take next? A) Suggest an epidural. B) Educate on the risks and benefits of pharmacologic interventions. C) Encourage the patient to ambulate in the room. D) Administer the prescribed dose of intravenous pain medication.

B) Educate on the risks and benefits of pharmacologic interventions. (By providing the patient with education and options, the nurse empowers her to make decisions about her body and labor and delivery experience.)

Which emotional expressions of "baby blues" are most common? A) Exhaustion and withdrawal B) Tearfulness and tiredness C) Anxiety and hopelessness D) Paranoia and insomnia

B) Tearfulness and tiredness (Women who are experiencing tearfulness and tiredness most likely have the "baby blues." These emotional expressions typically subside around 2 weeks after childbirth; however, feelings of tiredness may persist for many months.)

Which nursing interventions can help prevent abdominal distention and gas discomfort for a postpartum woman who delivered by cesarean birth? A) Administer oral (PO) narcotics. B) Encourage increased ambulation. C) Encourage increased oral (PO) fluid intake. D) Discourage the use of drinking straws. E) Encourage intake of carbonated beverages.

B) Encourage increased ambulation. (Increased ambulation stimulates peristalsis and helps prevent abdominal distention and gas for a woman who delivered by cesarean birth.) C) Encourage increased oral (PO) fluid intake. (Increased PO fluid intake stimulates peristalsis and helps prevent abdominal distention and gas for a woman who delivered by cesarean birth.) D) Discourage the use of drinking straws. (Women should be instructed to avoid straws to prevent abdominal distention and gas.)

Which solution can the nurse consider to encourage a positive birth experience for the patient and her support person following delivery? A) Offer to take the baby to the nursery to promote rest for the patient and the support person. B) Encourage the family to talk about the labor and delivery experience. C) Immediately after birth, remove amniotic fluid and vernix from the baby before placing it skin-to-skin with the mother. D) Provide all infants with pacifiers to promote infant and family rest.

B) Encourage the family to talk about the labor and delivery experience. (Encouraging the patient and the support person to talk freely about the birth experience is a very important way to help them make sense of the experience and to reflect on the positive and negative aspects of the labor and delivery.)

A nurse caring for a patient experiencing maternal exhaustion who desires a natural labor would implement which intervention to promote normal labor progress and decrease fatigue? A) Limit intravenous fluids (IV) to prevent overhydration. B) Encourage the patient to take a warm shower or bath. C) Insert an indwelling urinary catheter so the patient does not have to ambulate as frequently. D) Restrict visitation to given time intervals to allow for sleep.

B) Encourage the patient to take a warm shower or bath. (Hydrotherapy is an effective way to promote rest and relaxation in the laboring patient.)

Based on the nurse's understanding of normal breast changes in pregnancy, which response would a nurse provide to a pregnant patient who complains of "tumors" and "cancer" in her breasts? A) Explain that breast cancer is often triggered by pregnancy. B) Explain that nodular breast tissue is normal during pregnancy. C) Tell the woman she is exhibiting signs of altered coping related to the pregnancy. D) Inform the woman that nodular breast tissue is abnormal during pregnancy, so she needs a mammogram.

B) Explain that nodular breast tissue is normal during pregnancy. (Nodular breast tissue is often caused by the increase in estrogen and progesterone during pregnancy.)

Which method would the nurse recognize as a low-tech assessment of fetal well-being? A) Fetal aneuploidy screening B) Fetal kick counts C) Ultrasonography D) Biophysical profile assessment

B) Fetal kick counts (Fetal kick counts are an inexpensive, noninvasive method of fetal assessment.)

A nurse is caring for the following labor patients. Which patients would the nurse be prepared to monitor closely for signs of postpartum hemorrhage (PPH)? A) G1 P0000, delivered a 29-week fetal demise vaginally after 8 hours in labor B) G2 P1001, delivered a 4200-g neonate vaginally after 4 hours of labor C) G2 P0010, delivered a 3750-g neonate by cesarean section for breech presentation D) G4 P3003, delivered a 3500-g neonate by cesarean section with a placenta accreta] E) G3 P0200, delivered a 3900-g neonate vaginally after 36 hours in labor

B) G2 P1001, delivered a 4200-g neonate vaginally after 4 hours of labor (Fetal macrosomia (birth weight >4000 g) places a woman at higher risk for PPH after delivery due to an overstretched uterus.) D) G4 P3003, delivered a 3500-g neonate by cesarean section with a placenta accreta (A placenta accreta increases the risk for a woman to have a PPH due to the risk for retained placenta, which can lead to uterine atony.) E) G3 P0200, delivered a 3900-g neonate vaginally after 36 hours in labor (A prolonged labor, in this case 36 hours, increases a woman's risk for a PPH due to an overworked and "tired" uterus.)

The nurse midwife writes in the progress note that the baby is in the occiput posterior presentation. The nurse knows that which maternal position will help facilitate rotation of the fetal head? A) Side-lying B) Hands and knees C) Semi-recumbent D) Lithotomy

B) Hands and knees (Hands-and-knees positions are thought to be especially useful when a baby presents in the occiput posterior position. This is thought to pull the fetal back forward, allowing for better rotation of the fetal head. Other benefits of this position are similar to those of the upright position.)

Which assessment findings suggest excessive blood loss requiring immediate intervention for a postpartum patient who had a cesarean delivery? A) Firm fundus, midline B) Heart rate of 120 beats/min C) Blood pressure of 80/40 mm Hg D) Urinary output of 20 mL/hour E) Abdominal distension and severe pain

B) Heart rate of 120 beats/min (An elevated heart rate of 120 beats/min may be related to excessive blood loss/shock and requires immediate intervention.) C) Blood pressure of 80/40 mm Hg (A low blood pressure of 80/40 mm Hg may be related to excessive blood loss/shock and requires immediate intervention.) D) Urinary output of 20 mL/hour (Urinary output of less than 30 mL/hour may be related to excessive blood loss/shock and requires immediate intervention.) E) Abdominal distension and severe pain (Abdominal distension and severe pain may be related to internal bleeding and require immediate intervention.)

Which nursing action is appropriate when a 37-week pregnant woman complains of "getting lightheaded and dizzy" while having her fetal heart tones checked? A) Assist the woman to sit up. B) Help the patient turn to her left side. C) Check the woman's blood pressure. D) Notify the health care provider.

B) Help the patient turn to her left side. (This woman is experiencing supine hypotension/vena cava syndrome. Helping the patient turn to her left side is the first intervention the nurse can perform to displace the uterus off of the vena cava and increase blood flow.)

A patient requests nonhormonal intrauterine birth control. The nurse prepares which device for insertion by the nurse midwife? A) The medroxyprogesterone injection B) The Copper T380A intrauterine device C) The levonorgestrel intrauterine system D) The etonogestrel implant

B) The Copper T380A intrauterine device (The Copper T380A intrauterine device (IUD) is a copper-containing IUD that contains no hormones.)

Which TORCH infections may be transmitted perinatally? A) Toxoplasmosis B) Herpes simplex virus C) Gonorrhea D) Varicella zoster E) Rubella F) Cytomegalovirus

B) Herpes simplex virus (Herpes simplex virus may be transmitted transplacentally, perinatally, or after birth.) C) Gonorrhea (Gonorrhea is transmitted perinatally, although last trimester ascending infection may occur.) D) Varicella zoster (Varicella zoster is transmitted transplacentally, perinatally, or after birth.) F) Cytomegalovirus (Cytomegalovirus may be transmitted transplacentally, perinatally, or after birth.)

Which risk factor identified by the nurse places the patient at risk for preterm labor (PTL)? A) Family history of cervical cancer B) Homelessness C) Hypertension D) History of birth of a child at 37 weeks

B) Homelessness (Social and environmental factors, such as inadequate or absent prenatal or dental care, maternal domestic violence episodes, maternal smoking, homelessness, and age and ethnicity are all associated with increasing a woman's risk for PTL.)

Which action would the nurse take if a pregnant patient opts out of prenatal human immunodeficiency virus (HIV) testing? A) Report the mother to child protective services. B) Inform the mother that the baby will likely be screened for HIV after birth. C) Collect the sample anyway. D) Request that the patient's sexual partners come in for screening.

B) Inform the mother that the baby will likely be screened for HIV after birth. (It is recommended that the neonate be screened after birth even if the mother has opted out of prenatal testing.)

Which interventions would the nurse implement when caring for a newborn postcircumcision? A) Carefully remove yellow crust over glans. B) Loosely attach diaper. C) Clean glans with warm water and soap. D) Carefully document urinary output. E) Monitor for bleeding and signs of infection

B) Loosely attach diaper. (The nurse would attach the diaper loosely to prevent pressure to the circumcision site.) D) Carefully document urinary output. (The nurse would document urinary output, which is an indicator of healing and normal urinary function.) E) Monitor for bleeding and signs of infection (The nurse would monitor for bleeding and/or signs of infection (redness, swelling, purulent drainage). Should these occur, the nurse would contact the health care provider.)

Which physiologic finding influences balance changes in a pregnant woman? A) Kyphosis B) Lordosis C) Diastasis recti D) Back pain

B) Lordosis (Lordosis is the excessive inward curvature of the spine that occurs as the growing fetus pulls the center of gravity forward. Typically, the spine will adjust to realign the pregnant woman's center of gravity; however, balance changes (including falling) may occur because of this physiologic change in pregnancy.)

During the fourth stage of labor, the nurse notes an increased amount of bleeding. The uterine fundus consistency is boggy. Which is the most appropriate initial nursing action? A) Alert the health care provider that the patient is experiencing postpartum hemorrhage. B) Massage the uterine fundus and observe for change in consistency. C) Administer uterotonic medications to increase uterine tone. D) Do nothing, as this is a normal finding in the fourth stage of labor and the tone is transient.

B) Massage the uterine fundus and observe for change in consistency. (This is the most appropriate initial nursing action. Often uterine massage will cause a change in uterine tone from boggy to firm and will decrease vaginal lochia flow.)

A pregnant woman presents with clumpy, white vaginal discharge, vulvar pruritus, and burning. Which treatment would the nurse expect for this patient? A) Clindamycin B) Miconazole C) Metronidazole D) Imiquimod

B) Miconazole (Clumpy, white vaginal discharge with vulvar pruritus and burning is associated with candidiasis and should be treated with the appropriate antifungal, such as miconazole.)

Which interventions are most appropriate when caring for a patient admitted for preeclampsia on magnesium sulfate therapy? A) Encourage increased PO fluid intake. B) Monitor strict intake and output. C) Place protective padding on the bed's side rails. D) Assist patient with ambulating to the bathroom instead of using a bedpan. E) Ensure that calcium gluconate is immediately available.

B) Monitor strict intake and output. (Strict intake and output are essential for a patient receiving magnesium sulfate due to the risk for fluid volume overload and because of the need for monitoring renal function.) C) Place protective padding on the bed's side rails. (This patient is at risk for seizures, and padding the side rails helps prevent injury to the patient in the event of a seizure.) E) Ensure that calcium gluconate is immediately available. (Calcium gluconate is administered in the event of magnesium sulfate toxicity, and it should be immediately available.)

A patient is a G2/P0 at 32 weeks pregnant and experiencing regular contractions. She has a multifetal gestation pregnancy, a history of preterm birth, and has had recurrent bacterial vaginosis throughout the pregnancy. Her BMI is 22, she is 30 years' old, and she is a former smoker who quit two years ago. Of the data provided, which are risk factors for preterm labor? A) Age B) Multifetal gestaation C) Recurrent bacterial vaginosis D) Former smoker status E) BMI 22

B) Multifetal gestaation (The risk for preterm labor increases when a woman experiences a multifetal pregnancy because the uterus is overdistended.) C) Recurrent bacterial vaginosis (Recent studies have shown a link between vaginal infections in pregnancy and preterm labor.)

A woman received 25 mg of meperidine intravenously 1 hour before delivery. Which drug would the nurse have readily available? A) Ibuprofen B) Naloxone C) Nalbuphine D) Pitocin

B) Naloxone (Naloxone is the antagonist for opioid medications and should be available for administration to the neonate who exhibits signs of respiratory depression after birth.)

Which patient condition is a common breastfeeding cue in the first 24 hours following delivery? A) Mastitis B) Nipple soreness C) Plugged milk ducts D) Everted nipples

B) Nipple soreness (Nipple soreness due to poor latch or positioning is one of the most common complaints of women who are breastfeeding in the first 24 hours following delivery.)

Which nursing findings are concerning when assessing the breasts and nipples of a postpartum woman? A) Nipples are pink with intact skin. B) Nipples are pink with a blister line. C) Breasts are symmetric. D) Breasts are red and firm. E) Breasts are soft and nontender.

B) Nipples are pink with a blister line. (The nurse would not anticipate that a postpartum mother's nipples would be pink with blisters. Nipple damage is often the result of a poor latch by a breastfeeding infant.) D) Breasts are red and firm. (The nurse would not anticipate that a postpartum mother's breasts would be red and firm. This finding may be a symptom of mastitis.)

Which probabilities are associated with a man with hemophilia A having a child with a woman who does not have the disease and who is not a carrier? A) All of his sons will be affected. B) None of his sons will be affected. C) All of his daughters will be carriers. D) None of his daughters will be carriers.

B) None of his sons will be affected. (None of his sons would be affected because they receive only the Y chromosome from him, and all of his daughters will be carriers.) C) All of his daughters will be carriers. (All of his daughters will be carriers, and none of his sons would be affected because they receive only the Y chromosome from him.)

Which nursing comfort measures are recommended for a woman in her seventh month of pregnancy complaining of nasal congestion and occasional epistaxis? A) Warm compresses over the nasal area B) Normal saline nose spray C) Warm-mist humidifier in the room where she sleeps D) Oxymetazoline nasal spray

B) Normal saline nose spray (Nasal congestion and occasional nosebleeds are a common discomfort of pregnancy and are due to an elevated estrogen level. Normal saline nose spray may temporarily alleviate the congestion and lubricate the nasal passages.)

A multiparous woman is admitted to the postpartum unit after a rapid labor and birth of a 4000-g infant. Her fundus is boggy, lochia is heavy, and vital signs are unchanged. The nurse has the woman void and massages her fundus, but the patient's fundus remains difficult to find and the lochia remains bright red and heavy. Which action would the nurse take next? A) Vigorously massage the fundus B) Notify the health care provider C) Recheck the vital signs D) Insert an indwelling catheter

B) Notify the health care provider (Treatment of a postpartum hemorrhage requires the collaboration of the nurse and the health care provider. The nurse should remain with the patient and call the health care provider.)

A nurse who is called to a patient's room notes that the patient's cesarean incision has separated. Which action is the highest priority for the nurse to perform? A) Flush the wound with normal saline. B) Notify the health care provider. C) Elevate the head of the patient's bed to a high Fowler's position. D) Flex the patient's knees.

B) Notify the health care provider. (The nurse should notify the health care provider immediately for orders on how to treat this wound dehiscence.)

When evaluating a patient with suspected preterm premature rupture of membranes and preterm labor, the nurse recognizes which cues as signs of preterm labor? A) Dysuria and urinary frequency B) Pain and discomfort in the upper inner thighs C) Intermittent or constant lower back pain D) A Sensation that the fetus is frequently "balling up" E) The perception of decreased fetal movements F) Diarrhea

B) Pain and discomfort in the upper inner thighs (Pain in the upper inner thighs or vulva are often reported by women experiencing preterm labor.) C) Intermittent or constant lower back pain (Preterm labor contractions are often perceived differently than term labor contractions. Women often complain of intermittent or constant lower back pain.) D) A sensation that the fetus is frequently "balling up" (The fetus balling up is actually how the mother is perceiving the uterine contraction and is likely a uterine contraction rather than fetal movement.) F) Diarrhea (Abdominal cramping with or without diarrhea is sometimes a cue experienced by women with preterm labor.)

Occiput posterior refers to which component of the birth process? A) Powers B) Passenger C) Passage D) Psyche

B) Passenger (Occiput posterior refers to the fetal presentation within the female pelvis. The occiput anterior position is the most favorable for vaginal birth.)

Which pregnancy-related condition is related to both maternal and fetal influences? A) Amniotic banding B) Polyhydramnios C) Nuchal cord D) Preeclampsia

B) Polyhydramnios (Polyhydramnios can happen without a known cause. However, maternal diabetes can also lead to polyhydramnios. Because the fetus creates amniotic fluid, the condition is considered both maternal and fetal.)

The nurse notes recurrent late decelerations that do not respond to intrauterine resuscitation on the fetal monitor attached to a patient with a concealed placental abruption. Which order would the nurse anticipate? A) Administer pain medication as prescribed. B) Prepare for emergency cesarean birth. C) Turn the patient to the left lateral position. D) Palpate the fundal tone.

B) Prepare for emergency cesarean birth. (This fetus is demonstrating signs of hypoxia leading to hypoxemia, likely as a result of an advancing abruption. Delivery should be expedited to prevent maternal/fetal morbidity and mortality.)

Which cardiovascular changes that occur in fetal circulation occur after birth? A) Pressure decreases in the left atrium. B) Pressure decreases in the right atrium. C) The three fetal shunts constrict or close. D) Pressure in the aorta and pulmonary artery increases. E) Blood flow from the pulmonary artery to the aorta reverses.

B) Pressure decreases in the right atrium. (After birth, pressure in the right atrium decreases.) C) The three fetal shunts constrict or close. (After birth, the fetal circulatory shunts are not needed because the lungs oxygenate blood; as a result, the shunts close.) E) Blood flow from the pulmonary artery to the aorta reverses. (After birth, pressure in the aorta rises as pressure in the pulmonary artery falls. This causes the direction of blood flow through the ductus arteriosus to reverse, from the aorta to the pulmonary artery.)

Which statements explain how plasma volume returns to baseline after delivery? A) Increased oxytocin secretion accelerates fluid depletion. B) Profuse sweating aids in decreasing plasma volume levels. C) Sodium retention aids in the diaphoresis of excess plasma volume. D) Increased urinary output promotes the excretion of excess plasma volume. E) Decreased aldosterone hormone levels promote diuresis of excess plasma volume.

B) Profuse sweating aids in decreasing plasma volume levels. (Diaphoresis, or profuse sweating, especially at night, aids in the depletion of excess plasma volume after birth.) D) Increased urinary output promotes the excretion of excess plasma volume. (Increased urinary output (diuresis) promotes the excretion of excess plasma volume after birth. Diuresis is caused by decreased aldosterone, decreased oxytocin, and decreased sodium retention after delivery.) E) Decreased aldosterone hormone levels promote diuresis of excess plasma volume. (Decreased aldosterone hormone levels promote diuresis and depletion of excess plasma volume after birth.)

The nurse is caring for a patient who had a forceps delivery that caused a perineal hematoma. Which nursing intervention is most appropriate? A) Administer topical analgesic ointment as prescribed. B) Provide the patient with an ice pack and educate her about its use. C) Encourage the patient to lie on her side as much as possible until the injury heals. D) Educate the patient about the fact that the hematoma was caused by the introduction of forceps.

B) Provide the patient with an ice pack and educate her about its use. (Ice packs can help relieve the pain of hematomas or lacerations after operative delivery by causing vasoconstriction and decreasing blood flow to the area, as well as decreasing edema. Patients should be guided toward use for the first 12 hours, followed by intermittent use.)

Which term best describes the transition between childhood and sexual maturity? A) Menarche B) Puberty C) Menopause D) Menstruation

B) Puberty (Puberty is a broad term denoting the transition between childhood and sexual maturity.)

The nurse caring for a pregnant patient with gestational diabetes at 33 weeks gestation assesses the fundal height at 36 cm. Which actions by the nurse are correct regarding this finding? A) Assure the patient that the fetus is growing appropriately for its age. B) Recommend that the health care provider order an ultrasound. C) Educate the patient on the importance of limiting weight gain for the remainder of the pregnancy. D) Perform a fetal nonstress test and report the findings to the health care provider. E) Assist the patient with scheduling a cesarean delivery at 37 weeks.

B) Recommend that the health care provider order an ultrasound. (Recommending that the health care provider order an ultrasound is an appropriate action. Amniotic fluid volume and fetal size should be monitored for this patient.) D) Perform a fetal nonstress test and report the findings to the health care provider. (Gestational diabetics require additional fetal surveillance as a result of the high-risk nature of the pregnancy. When assessment findings are abnormal, fetal well-being should be assessed. A nonstress test is part of a biophysical profile and gives the health care provider an insight into fetal well-being.)

For which reasons would patient education focus on smoking cessation during pregnancy? A) Risk for low blood sugar B) Risk for premature birth C) Risk for congenital anomalies D) Risk for increased fetal circulation E) Risk for low birth weight

B) Risk for premature birth (Smoking during pregnancy is related to preterm labor and low birth weight, both of which are associated with other neonate complications. It is crucial for patients to be educated about the potential risks of smoking.) E) Risk for low birth weight (Smoking during pregnancy is related to preterm labor and low birth weight, both of which are associated with other neonate complications. It is crucial for patients to be educated about the potential risks of smoking.)

Maternal role attainment is a process that spans from pregnancy through which time period? A) Immediately after birth, when the mother can practice her newborn care skills B) Several months after delivery. C) Until her first pediatrician visit postdischarge D) After 1 year postpartum

B) Several months after delivery (Maternal role attainment spans from pregnancy through several months postpartum.)

The nurse understands that which conditions are risks for a uterine inversion? A) Precipitous delivery B) Short umbilical cord C) Polyhydramnios D) Fundal implantation of the placenta E) Prolonged labor

B) Short umbilical cord (A short umbilical cord is a risk factor for a uterine inversion because it places extra traction on the cord during delivery, which could pull the placenta and invert the uterus.) D) Fundal implantation of the placenta (Fundal implantation of the placenta is a risk factor for a uterine inversion; as traction is placed on the umbilical cord during delivery, there is a higher risk to invert the uterus.) E) Prolonged labor (A prolonged labor is a risk factor for uterine inversion because a prolonged labor creates an overworked and "tired" uterus, which makes it vulnerable to inversion after delivery.)

An 18-year-old presents with a history of hip pain and is found to have avascular necrosis. Which genetic disorder may be present? A) Cystic Fibrosis B) Sickle Cell Disease C) Tay-Sachs Disease D) Edwards Syndrome

B) Sickle Cell Disease (Sickle cell disease is characterized by avascular necrosis, frequent painful crises, and acute chest syndrome.)

Which genetic disorders can be understood using Punnett squares? A) Polygenic Disorder B) Single Gene Disorder C) Chromosomal Disorder D) Autosomal Recessive Disorder E) Autosomal Dominant Disorder

B) Single Gene Disorder (Single gene disorders are characterized by changes in one gene and can be understood using Punnett squares.) D) Autosomal Recessive Disorder (Autosomal recessive disorders are characterized by changes in single genes and can be understood using Punnett squares.) E) Autosomal Dominant Disorder (Autosomal dominant disorders are characterized by changes in single genes and can be understood using Punnett squares.)

A woman is seeking a long-term contraceptive method that does not require taking daily medication or applying a device before intercourse. Which contraceptive methods would the nurse recommend? A) Lea's Shield B) Single-rod etonogestrel implant C) Contraceptive sponge D) Intrauterine device (IUD) E) Combined oral contraceptives (COCs)

B) Single-rod etonogestrel implant (The single-rod etonogestrel progestin implant can provide up to 3 years of contraception after insertion. Because it does not require daily medication or the application of a device before intercourse, it would be an appropriate method of contraception for this woman.) D) Intrauterine device (IUD) (IUDs provide either 5 years (hormonal IUD) or 10 years (copper IUD) of contraception after insertion. Because they do not require daily medication or the application of a device before intercourse, this would be an appropriate method of contraception for this woman.)

Which assessment data would support a diagnosis of pulmonary embolism (PE)? A) Calf pain with dorsiflexion of the foot B) Sudden onset of pleuritic chest pain and dyspnea C) Left-sided chest pain and diaphoresis D) Bilateral crackles and low-grade fever

B) Sudden onset of pleuritic chest pain and dyspnea (The most common signs of a PE are sudden onset of pleuritic chest pain when taking a deep breath and dyspnea.)

Which physical change indicates impending ovulation? A) Necrotic endometrium B) Thin, stringy cervical mucus C) Thickened endometrial secretions D) Dilation of endometrial blood vessels

B) Thin, stringy cervical mucus (Thin, stringy cervical mucus aids sperm entry into the uterus and also indicates impending ovulation.)

Which chromosomal abnormalities result in a form of Down syndrome? A) Trisomy 18 B) Trisomy 21 C) Polysomy X D) Monosomy X E) Translocation

B) Trisomy 21 (Down syndrome is most often characterized by three chromosomes, rather than the normal two, in the 21 position. This is also known as trisomy 21.) E) Translocation (Some forms of Down syndrome may have part of a chromosome 21 attached to another chromosome, which is an example of translocation.)

A G5/P4 laboring patient with suspected fetal intrauterine growth restriction has just experienced spontaneous rupture of membranes. On examination, the nurse notes that the cervix is dilated 3 cm and is 70% effaced and that the fetal station is 0. Fetal heart tracing shows recurrent and severe variable decelerations. Which explanation is the most likely cause for this change in fetal heart rate? A) High fetal station B) Umbilical cord prolapse C) Fetal intrauterine growth restriction D) Grand multiparity

B) Umbilical cord prolapse (Recall that umbilical cord prolapse may be "hidden" or otherwise not palpable during a cervical or vaginal examination. This patient has a very small fetus, and she just experienced rupture of membranes, both of which are cues indicating an increase in the risk for umbilical cord prolapse. Variable decelerations and bradycardia also accompany umbilical cord prolapse.)

Which classic findings would the nurse identify in a patient with placental abruption (abruptio placentae)? A) Absent vaginal bleeding B) Uterine irritability with low-intensity contractions C) Severe abdominal cramping D) Painless vaginal bleeding E) Boardlike abdomen

B) Uterine irritability with low-intensity contractions (The uterus becomes irritable as the placenta separates prematurely from the uterine wall.) C) Severe abdominal cramping (Pain is present as the placenta separates prematurely from the uterine wall.) E) Boardlike abdomen (Increased uterine tone is characteristic of placental abruption.)

Which interventions would the nurse implement for a breastfed newborn who has lost 8% of his or her birth weight? A) Recommend a feeding schedule of every 3 to 4 hours. B) Weigh the infant daily using same scale. C) Evaluate feeding technique and positioning. D) Demonstrate waking techniques for feeding a sleepy baby. E) Evaluate the newborn suck and latch.

B) Weigh the infant daily using same scale. (Weighing the infant daily at the same time of day with the same scale ensures accurate measurement and tracking of weight loss.) C) Evaluate feeding technique and positioning. (The nurse would evaluate the mother's feeding technique and positioning to assist with establishing effective feeding.) D) Demonstrate waking techniques for feeding a sleepy baby. (The nurse would demonstrate techniques for waking a sleepy baby to encourage effective breastfeeding.) E) Evaluate the newborn suck and latch. (The nurse would evaluate the newborn suck and latch to ensure effective breastfeeding.)

Place the changes in female sexual maturation in the typical order of occurrence. Axillary hair growth; Breast development; Menstruation; Pubic hair development

Breast development Pubic hair development Axillary hair growth Menstruation

A woman says, "My husband and I will be trying to have a baby soon." Which statement by the woman demonstrates an understanding of the growth that occurs during the embryonic period? A) "If I eat more, then the baby's structures will grow larger." B) "By week 8, all organs are formed, but none are functioning." C) "All of the major organs for my baby are formed in 8 weeks." D) "In the beginning, the fetal brain and heart are the only organs that grow rapidly."

C) "All of the major organs for my baby are formed in 8 weeks." (The woman has an accurate understanding that basic structures of all major body organs are developed during the embryonic period. By the end of the eighth week, all major organ systems are in place, and many are functioning in a simple way.)

Which statement indicates a need for further teaching regarding cesarean delivery? A) "I might need a blood transfusion after the surgery." B) "Having a C-section might make my hospital stay longer." C) "Because I am not having a vaginal birth, my babies can't be harmed during delivery." D) "Depending on how they do the incision, I may be able to have a vaginal delivery if I become pregnant again."

C) "Because I am not having a vaginal birth, my babies can't be harmed during delivery." (Cesarean delivery carries risk for injury to the newborn, such as bruising, fractures, or other trauma.)

Which nursing education is appropriate for a pregnant patient when discussing breast changes? A) Increased relaxin levels may lead to darkening of the areola. B) Secretion of colostrum is normal in the first trimester. C) "Bumps" on the nipples are normal. D) Breasts lose sensitivity during pregnancy.

C) "Bumps" on the nipples are normal. (Montgomery glands (small bumps on nipples) become more prominent and may produce a small amount of breast milk or a natural, oily substance that cleans and lubricates the nipple and areola.)

Which statement by a pregnant woman would suggest that she needs further teaching about the purpose of childbirth education classes? A) "Childbirth preparation programs will increase my sense of control." B) "Childbirth preparation programs will help prepare my support person to help during labor." C) "Childbirth preparation programs guarantee a pain-free childbirth." D) "Childbirth preparation programs teach distraction techniques."

C) "Childbirth preparation programs guarantee a pain-free childbirth." (This is not an expected outcome of a childbirth preparation program. No amount of preparation can guarantee a pain-free birth.)

The instructor is discussing the embryonic period with a nursing student. Which statement by the student nurse would indicate a need for additional teaching? A) "The chromosomes carry the basic instructions for development." B) "Signaling to stimulate change may occur between two tissues." C) "Development occurs from the peripheral direction inward." D) "Early cells begin as specialized cells but later develop the ability to become other cells."

C) "Development occurs from the peripheral direction inward."Development occurs from the center outward (center-to-peripheral direction).

Which parental statement indicates a need for further sleep safety education? A) "My baby sleeps on her back." B) "I dress my baby in a sleep sack for sleep." C) "I cover my baby with a warm quilt at night." D) "My baby sleeps in a bassinet next to my bed."

C) "I cover my baby with a warm quilt at night." (Babies should not use heavy quilts or blankets due to the risk for suffocation. This statement indicates a need for further education.)

Which parental statement would the nurse interpret as indicating a need for further education about newborn body processes? A) "My baby's breathing might be variable and irregular." B) "I will dress my baby similar to how I am dressed except with an additional blanket." C) "I will check my baby's temperature in his bottom when he is sick." D) "My baby should have at least six wet diapers a day."

C) "I will check my baby's temperature in his bottom when he is sick." (Rectal temperatures are not the preferred method of measuring temperature because there is a risk for injury. Parents should measure their baby's temperature by placing the thermometer high in the armpit (without causing pain) and holding the arm down. This parental statement indicates to the nurse that further education is needed.)

The nurse administered magnesium sulfate to a woman experiencing preterm labor, following a health care provider prescription. Which assessment would concern the nurse? A) Urine output of 50 mL B) Oxygen saturation of 95% C) 10 respirations per minute D) Bronchial sounds heard over the body of the sternum

C) 10 respirations per minute (The nurse assesses the woman who has been administered magnesium sulfate for respirations. Ten respirations per minute is a common hospital criterion to discontinue magnesium sulfate therapy.)

A nurse is reviewing discharge teaching with a patient who has a urinary tract infection (UTI). Which statements by the patient indicate understanding of the teaching? A) "I will perform perineal care and apply a perineal pad in a back-to-front direction." B) "I will drink cranberry and prune juices to make my urine more acidic." C) "I will drink large amounts of water to flush the bacteria from my urinary tract." D) "I will go back to breastfeeding after I have finished taking the antibiotic." E) "I will take analgesics for any discomfort."

C) "I will drink large amounts of water to flush the bacteria from my urinary tract." (Forcing fluids, especially water, is important to maintain hydration and to flush the urinary system of any bacteria.) E) "I will take analgesics for any discomfort." (The woman with a UTI will often complain of fever and urinary spasms. Taking an analgesic is recommended when she has a fever or discomfort.)

The nurse recognizes that the patient understands the teaching when she makes which statement? A) "Premonitory labor signs include rupture of membranes." B) "I will know I'm in labor when I lose my mucous plug." C) "Increasing clear vaginal secretions could mean labor will begin soon." D) "Bright red bleeding is considered bloody show and means labor will begin soon."

C) "Increasing clear vaginal secretions could mean labor will begin soon." (This is a premonitory sign of labor and means labor could begin soon.)

Which statement does the nurse use to describe to the patient's partner why opioid analgesics are being administered? A) "Opioid analgesics prevent nausea." B) "Opioid analgesics remove the pain of labor." C) "Opioid analgesics help the laboring woman relax between contractions." D) "Opioid analgesics reduce respiratory depression for the laboring woman."

C) "Opioid analgesics help the laboring woman relax between contractions." (The nurse explains that opioid analgesics affect the perception of pain, allowing the laboring woman to relax during contractions.)

A woman is reluctant to perform vulvar self-examinations and asks if they are necessary because she has no family history of reproductive cancers and is not sexually active. What patient education is appropriate for this woman? A) "If you are not sexually active, monthly examinations are not necessary." B) "If your Papanicolaou (Pap) test is normal, you only need to perform self-examination once every 3 months." C) "Performing self-examination now will help you recognize any abnormal changes that may occur in the future." D) "Because you do not have any family history of reproductive cancers, monthly examinations are not necessary."

C) "Performing self-examination now will help you recognize any abnormal changes that may occur in the future." (Vulvar self-examination should be performed regardless of family history for the woman to establish a baseline appearance and detect signs of infection or abnormal growth in the future.)

Which advice would the nurse provide a woman regarding the "baby blues" and postpartum depression as part of discharge teaching? A) "Stay home and avoid outside activities to ensure adequate rest." B) "Be certain that you are the only caregiver for your baby to facilitate infant attachment." C) "Talk about your feelings and seek help if needed." D) "Keep feelings of sadness and adjustment to your new role to yourself."

C) "Talk about your feelings and seek help if needed." (The "baby blues" occurs in almost 85% of postpartum women. This topic can carry feelings of shame, fear, and anxiety. Women should be encouraged to seek help and talk about their feelings, not hold them in.)

Which statement by a postpartum woman would require the most immediate attention by the nurse? A) "I am so tired I do not know how I am going to make it through the day." B) "I am not hungry. I am going to skip lunch today." C) "The lactation consultant tried to steal my baby." D) "I am never going to feel like myself again."

C) "The lactation consultant tried to steal my baby." (Paranoia and hallucinations could indicate postpartum psychosis, a postpartum mood disorder that requires immediate medical attention.)

The nurse knows that patient education has been effective when the patient makes which statement about the difference between a tocodynamometer and an intrauterine pressure catheter (IUPC)? A) "Only the tocodynamometer shows my uterine activity." B) "The tocodynamometer is much more accurate than the IUPC." C) "The tocodynamometer is positioned outside my body, while the IUPC is positioned inside my body." D) "The tocodynamometer will be connected to my bedside monitor, but the IUPC will not."

C) "The tocodynamometer is positioned outside my body, while the IUPC is positioned inside my body." (Whereas the tocodynamometer is an external monitoring device, the IUPC is an internal monitoring device.)

A patient is concerned about the baseline variability in the heart rate of her fetus. Which responses by the nurse describe the significance of baseline variability to the patient? A) "Variability is an artifact." B) "Variability is a periodic pattern." C) "Variability demonstrates that there is adequate oxygenation of the fetus." D) "Variability suggests that the fetus is able to adapt to the labor process." E) "Variability indicates that the fetus has no congenital abnormalities."

C) "Variability demonstrates that there is adequate oxygenation of the fetus." (Adequate oxygenation of the fetus, demonstrated by variability, is necessary, and therefore variability is significant.) D) "Variability suggests that the fetus is able to adapt to the labor process." (Variability is significant because its presence indicates that the autonomic nervous system is intact, allowing the fetus to adapt to the normal stress of labor.)

Which education would the nurse provide a mother who reports latch difficulty with breastfeeding? A) "The newborn is likely sleepy. You should wait until the baby is fully awake." B) "Use a breast pump for a few minutes before feedings to increase nipple size." C) "Wait until the mouth is open wide to insert the nipple into the newborn's mouth." D) "Apply ointment to the nipple. It has a sweet taste and will prompt the newborn to breastfeed."

C) "Wait until the mouth is open wide to insert the nipple into the newborn's mouth." (The mother should be taught that if the mouth is not open wide before the nipple is inserted, it is more difficult for the newborn to get sufficient milk and can cause nipple pain.)

Which response would the nurse provide the patient who asks why oxygen is being given after the nurse identifies a nonreassuring fetal heart rate? A) "I will call the health care provider to discuss the new care plan." B) "We need to increase the perfusion of the baby's placenta." C) "We need to increase your oxygen, which will increase the baby's oxygen." D) "Don't worry. This happens all of the time, and everything is fine."

C) "We need to increase your oxygen, which will increase the baby's oxygen." (The nurse provides a simple and direct explanation for the intervention being provided.)

Which situation puts a family at risk for poor adaptation after birth? A) A mother who labored for 8 hours B) A mother who had a planned, repeat cesarean delivery C) A mother who delivered at 32 weeks gestation D) A mother who had a spontaneous vaginal birth of an infant with 9/9 Apgar score

C) A mother who delivered at 32 weeks gestation (A mother who delivered at 32 weeks gestation would put the family at risk for poor adaptation to birth. A preterm delivery is an unexpected birth event, and the newborn will need to be placed in the neonatal intensive care unit (NICU).)

The nurse is caring for a patient in the second stage of labor. Which patient condition is most likely to result in the need for an episiotomy? A) A patient with a history of perineal laceration B) A patient receiving oxytocin for induction of labor C) A patient whose fetus is experiencing shoulder dystocia D) A patient who had an episiotomy during a previous delivery

C) A patient whose fetus is experiencing shoulder dystocia (Shoulder dystocia is an indication for episiotomy because it is necessary to allow as much room as possible for the delivery of the shoulder.)

When counseling a patient on the warning signs of complications associated with contraception, the nurse recognizes which patient cue as a warning sign for both oral contraceptive pills (estrogen-containing contraception) and intrauterine devices? A) Fever or chills B) Headaches C) Abdominal pain D) Abnormal vaginal discharge

C) Abdominal pain (Abdominal pain can act as a warning sign of both intrauterine devices (PAINS [period late, abnormal spotting or bleeding; abdominal pain, pain during intercourse; infection exposure, abnormal vaginal discharge; not feeling well, fever, chills; string missing, shorter or longer]) and oral contraceptive pills (ACHES [abdominal pain, chest pain, headaches, eye problems, swelling and/or aching in legs and thighs]).)

A postpartum mother informs the nurse that she is disappointed in the gender of her baby. Which nursing intervention would be most appropriate to support adaptation after birth? A) Instruct the patient not to disclose this to others. B) Remove the newborn from the mother's room and notify the health care provider. C) Acknowledge the patient's feelings and provide emotional support. D) Inform the patient that she should be grateful for a healthy newborn baby.

C) Acknowledge the patient's feelings and provide emotional support. (Acknowledging the patient's feelings and providing emotional support will support adaptation after birth.)

A woman tests positive for group B streptococcus (GBS) at 36 weeks gestation before a vaginal birth. Which interventions would the nurse include in the plan of care? A) Retest for GBS upon admission to labor and delivery. B) Instruct the patient to use condoms until the infection clears. C) Administer antibiotics during labor until delivery as prescribed. D) Monitor the neonate for signs and symptoms of respiratory distress. E) Inspect the perineum carefully for lesions upon admission for labor and delivery.

C) Administer antibiotics during labor until delivery as prescribed. (Antibiotics are administered in labor to help prevent transmission to the neonate during delivery.) D) Monitor the neonate for signs and symptoms of respiratory distress. (Signs of respiratory distress accompany neonatal GBS infection.)

Which order during labor augmentation would cause the nurse to question the health care provider? A) Administer oxytocin in lactated Ringer solution per protocol. B) Administer oxytocin in normal saline per protocol. C) Administer oxytocin in dextrose 10% per protocol. D) Administer oxytocin in water per protocol.

C) Administer oxytocin in dextrose 10% per protocol. (Using hypertonic solutions such as dextrose 10% increases the risk for water intoxication because it increases the antidiuretic effects of oxytocin.)

A nurse is caring for a hospitalized patient who is receiving intravenous heparin for deep vein thrombosis (DVT). The patient begins to vomit blood. After the heparin has been stopped, which action would the nurse take next? A) Administer vitamin K1. B) Measure the emesis. C) Administer protamine sulfate. D) Perform the Homans sign.

C) Administer protamine sulfate. (When clinical circumstances (bleeding) require reversal of heparinization, protamine sulfate (1% solution) by slow infusion will neutralize heparin sodium. No more than 50 mg should be administered very slowly in any 10-minute period. Each milligram of protamine sulfate neutralizes approximately 100 USP heparin units.)

Which considerations would the nurse recognize for a pregnant patient with human immunodeficiency virus (HIV)? A) A second test toward the end of the third trimester is not recommended. B) Vaginal delivery is always contraindicated. C) Antiretroviral therapy for the mother and neonate helps reduce transmission at birth. D) Testing all pregnant women for HIV at the beginning of pregnancy is a best practice. E) Breastfeeding should be avoided. F) An elective cesarean birth at 38 weeks helps prevent transmission to the neonate.

C) Antiretroviral therapy for the mother and neonate helps reduce transmission at birth. (The risk for transmission decreases from 30% to less than 2% with antiretroviral therapy.) D) Testing all pregnant women for HIV at the beginning of pregnancy is a best practice. (All women, regardless of history, should be tested for HIV in the beginning of pregnancy.) E) Breastfeeding should be avoided. (Breastfeeding is contraindicated with maternal HIV infection.) F) An elective cesarean birth at 38 weeks helps prevent transmission to the neonate. (Elective cesarean birth has been shown to reduce the rate of neonatal HIV transmission.)

A patient reports a fishy, greenish, malodorous vaginal discharge after intercourse. The nurse understands these findings to be classic symptoms of which condition? A) Genital warts (condyloma) B) Chlamydia C) Bacterial vaginosis (Gardnerella) D) Vaginal yeast infections (Candidiasis)

C) Bacterial vaginosis (Gardnerella) (The number one complaint for women with bacterial vaginosis is a fishy, greenish, thin discharge that is worse after intercourse.)

The nurse understands that which concurrent condition during pregnancy is the leading cause of maternal mortality in the United States today? A) Cytomegalovirus (CMV) B) Human immunodeficiency virus (HIV) C) Cardiovascular disease D) Anemia secondary to blood loss

C) Cardiovascular disease (Cardiovascular disease is now the leading cause of maternal mortality in the United States today.)

Which health risk is associated with perimenopause? A) Reproductive cancers B) Flushing C) Cardiovascular disease D) Vaginal dryness

C) Cardiovascular disease (The risk for cardiovascular disease increases due to the decrease in estrogen production that occurs during perimenopause.)

Which clinical conditions must be met before placing a fetal scalp monitor? A) Patient should not be in labor B) Absence of fetal scalp hair C) Cervix should be at least 2 cm dilated D) The patient's membranes must be ruptured E) High fetal presentation

C) Cervix should be at least 2 cm dilated (The fetal scalp monitor may be placed once the patient's cervix has dilated to at least 2 cm. The cervix must be dilated enough to allow contact with the fetal scalp.) D) The patient's membranes must be ruptured (The fetal scalp monitor can be used once the patient's membranes have ruptured to allow contact with the fetal scalp.)

The nurse is checking the pad of a patient recovering from a cesarean delivery who has called out, "I just had a large gush of blood!" The nurse does not note an abnormal amount of blood under the patient's legs. Which action would the nurse take next? A) Request that the health care provider evaluate the patient for possible postpartum hemorrhage. B) Administer the as-needed (PRN) dose of methylergonovine. C) Check for pooling of blood under the patient's buttocks and back. D) Explain that this can be normal and evaluate with the next scheduled fundal check.

C) Check for pooling of blood under the patient's buttocks and back. (Significant amounts of blood can pool under a patient's buttocks or even up the back when the patient is bedbound. It is important to immediately evaluate this patient's concern.)

A woman visiting the health care provider for a routine well-woman visit has a history of stroke and has right-sided hemiparesis, or weakness. The nurse anticipates that the results of which screening procedure may be affected by these symptoms? A) Papanicolaou (Pap) test B) Mammography C) Clinical breast examination (CBE) D) Thyroid-stimulating hormone (TSH)

C) Clinical breast examination (CBE) (During the CBE, the woman is instructed to lift both arms above the head. Hemiparesis will affect the woman's ability to lift both arms; therefore the results of the visual inspection made by the nurse will be affected.)

Which type of placenta previa would the nurse recognize as being most dangerous? A) Concealed previa B) Marginal previa C) Complete previa D) Partial previa

C) Complete previa (A complete previa is the most dangerous, as the placenta completely covers the cervical os, increasing the risk for injury and bleeding.)

Which signs would the nurse recognize as indicative of missed abortion? A) Vaginal bleeding B) Products of conception partially expelled C) Decrease in uterine size D) Absent fetal heart rate E) Subsiding nausea F) Absence of breast tenderness

C) Decrease in uterine size (As the fetus degenerates, the fundal height decreases.) D) Absent fetal heart rate (With missed abortion, there is no fetal heart tone.) E) Subsiding nausea (Signs of pregnancy begin to disappear after fetal death.) F) Absence of breast tenderness (Breast tenderness, an early sign of pregnancy, disappears after fetal death.)

Which condition is related to the increased risk for respiratory distress in neonates born to mothers with gestational diabetes? A) Hyperinsulinemia after birth B) Recurrent fetal hypoxia and subsequent increased production of erythrocytes C) Decreased fetal cortisol levels and therefore insufficient surfactant D) Decreased maternal magnesium levels

C) Decreased fetal cortisol levels and therefore insufficient surfactant (Surfactant helps the alveoli function properly after birth and promotes adequate ventilation in the newborn. When surfactant is insufficient, ventilation may be impaired.)

Which physiologic change in fetal circulation is triggered by clamping of the umbilical cord after birth? A) First breath B) Foramen ovale closure C) Ductus venosus constriction D) Ductus arteriosus constriction

C) Ductus venosus constriction (The ductus venosus constricts when blood flow from the umbilical cord stops as a result of clamping the cord.)

Which deceleration is considered a normal finding? A) Variable B) Prolonged C) Early D) Late

C) Early (Early decelerations are a result of fetal head compression and are considered a normal finding that is not associated with poor fetal status or outcomes.)

Which action taken by the patient will reduce discomfort during Leopold maneuvers and make fetal presenting parts easier to feel? A) Standing B) Lying prone C) Emptying the bladder D) Pushing or bearing down with contractions

C) Emptying the bladder (Emptying the bladder reduces discomfort during palpation and makes fetal parts easier to feel.)

The nurse is developing a standard care plan for a mother after a cesarean delivery. Which step would the nurse plan to implement? A) Maintain patient in a left-lateral recumbent position for the first 24 hours after the cesarean section. B) Teach the patient how to do sitz baths for pain on the second postoperative day. C) Encourage early, frequent ambulation after the surgery. D) Assess arterial blood gases during the first postoperative day.

C) Encourage early, frequent ambulation after the surgery. (It is crucial for patients who have had any surgery, including a cesarean section, to ambulate early and frequently and to use antithrombotic boots during bed rest.)

A woman is experiencing strong contractions every 1.5 to 2 minutes, feels rectal pressure, and has a large amount of bloody show. At which frequency would the nurse anticipate assessing the fetal heart rate and pattern? A) Every 30 to 45 minutes B) Every 5 to 10 minutes C) Every 15 to 30 minutes D) Every 45 to 60 minutes

C) Every 15 to 30 minutes (Based on the patient's signs and symptoms, she is in active labor, so it is recommended that the nurse assess the fetal heart rate and pattern every 15 to 30 minutes.)

Which statement explains how a 5-year-old girl could present with pubic hair growth? A) Estrogen is released prematurely by the anterior pituitary gland. B) Luteinizing hormone (LH) is released in large quantities from the ovaries. C) Gonadotropin-releasing hormone (GnRH) is released prematurely by the hypothalamus. D) Follicle-stimulating hormone (FSH) is released in large quantities from the hypothalamus.

C) Gonadotropin-releasing hormone (GnRH) is released prematurely by the hypothalamus. (A 5-year-old girl who presents with pubic hair growth is experiencing isosexual precocious puberty caused by premature release of GnRH.)

For which TORCH infections would the nurse anticipate administering immune globulin? A) Toxoplasmosis B) Gonorrhea C) Hepatitis B virus D) Varicella zoster E) Syphilis F) Rubella

C) Hepatitis B virus (Hepatitis B immune globulin may be administered to infants of HBsAg mothers.) D) Varicella zoster (Varicella zoster immune globulin may be administered to infants with varicella zoster.)

A laboring woman wishes to labor in the shower to alleviate pain. Which nonpharmacologic pain relief method is she applying? A) Massage B) Acupressure C) Hydrotherapy D) Relaxation

C) Hydrotherapy (Taking a shower is an example of hydrotherapy, a form of cutaneous stimulation, and it is a relaxation technique that can be very helpful for laboring women.)

The nurse notes fetal tachycardia and suspects that the patient may be dehydrated. Which nursing action is appropriate to address this nonreassuring finding? A) Consult with the dietician. B) Administer parenteral feeding. C) Increase the rate of intravenous (IV) saline administration. D) Provide the patient an oral (PO) electrolyte replacement.

C) Increase the rate of intravenous (IV) saline administration. (Tachycardia can be the result of maternal hypovolemia caused by dehydration. Increasing the rate of nonadditive intravenous fluids can improve placental perfusion by increasing maternal blood volume.)

A patient arrives at a birthing center in active labor. Her membranes are still intact, and the health care provider prepares to perform an artificial rupture of membranes (AROM). What will the nurse relay to the patient as the most likely outcome of the procedure? A) Less pressure on the cervix B) Decreased number of contractions C) Increased pressure on the cervix D) The need for more cervical exams

C) Increased pressure on the cervix (The rupture of the amniotic membranes releases the cushion provided by the amniotic sac and causes an increase in the pressure of the fetal head on the cervix. This increased pressure will often result in an increase in contractions and a decrease in the time for dilation and effacement of the cervix.)

Which uterine activity indicators does the intrauterine pressure catheter (IUPC) measure in mm HG? A) Frequency B) Duration C) Intensity D) Resting tone E) Variability

C) Intensity (Contraction intensity is measured by the IUPC in mm Hg and is 50 to 75 mm Hg during labor and up to 110 mm Hg with pushing during the second stage.) D) Resting tone (Resting tone is measured by the IUPC in mm Hg and is usually between 5 and 15 mm Hg.)

Using the Naegele rule, what is the estimated due date (EDD) for a woman who reports the first day of her last menstrual period (LMP) as September 20? A) May 30 B) June 20 C) June 27 D) July 3

C) June 27 (Naegele rule: LMP - 3 months + 7 days = EDD (June 27))

Through which mechanism does nonshivering thermogenesis generate heat in a newborn? A) Vasodilation B) Diminished metabolic activity C) Metabolism of brown fat D) Increased respiratory rate

C) Metabolism of brown fat (Nonshivering thermogenesis increases the newborn's body temperature by metabolizing brown fat.)

For which condition would the nurse teach the family that breastfeeding is contraindicated but feeding expressed breast milk is safe? A) Infant galactosemia B) Mother with human immunodeficiency virus (HIV) C) Mother with active tuberculosis D) Mother with active herpes simplex virus (HSV) lesions on the breast

C) Mother with active tuberculosis (While breastfeeding is contraindicated in mothers with active tuberculosis, expressed breast milk is considered safe. The nurse would teach this mother to feed her infant expressed breast milk.)

Which associated risk would a fertility specialist discuss with a couple who is considering the use of assisted reproductive technology (ART)? A) Weight loss B) Polycystic ovary syndrome C) Ovarian hyperstimulation D) Tubal adhesions

C) Ovarian hyperstimulation (Ovarian hyperstimulation, in which the ovaries become swollen and painful, is an excessive response to taking infertility medications, may occur with pharmacologic treatment for infertility, and is a risk associated with ART.)

Which signs and symptoms are commonly associated with endometritis? A) Flank pain B) Breast tenderness C) Pelvic pain D) Excessive lochia E) Low-grade fever for 24 hours

C) Pelvic pain (Pelvic pain is a common sign of endometritis because endometritis involves an infection of the uterine lining.) D) Excessive lochia (Excessive lochia is a common sign of endometritis because endometritis involves an infection of the uterine lining.)

The labor nurse is evaluating the patient's most recent 10-minute segment on the monitor strip and notes a late deceleration. Which explanations most likely explains this occurrence? A) Spontaneous fetal movement B) Compression of the fetal head C) Placental abruption D) Nuchal cord around the baby's neck E) Vena cava syndrome

C) Placental abruption (Late decelerations are almost always caused by uteroplacental insufficiency, the result of uterine tachysystole, maternal hypotension, epidural or spinal anesthesia, intrauterine growth restriction (IUGR), intraamniotic infection, or placental abruption.) E) Vena cava syndrome (Late decelerations are almost always caused by uteroplacental insufficiency, the result of uterine tachysystole, maternal hypotension, epidural or spinal anesthesia, intrauterine growth restriction (IUGR), intraamniotic infection, or placental abruption.)

Which patient data would the nurse validate as part of routine preparation for cesarean delivery to determine adequate fetal maturity? A) Rh-positive antibody test performed 4 weeks before surgery B) Negative rubella titer performed 6 weeks before surgery C) Positive serum pregnancy test performed at least 36 weeks before surgery D) High serum alpha-fetoprotein (AFP) level performed 26 weeks before surgery E) Ultrasound examination between 6 and 11 weeks of pregnancy

C) Positive serum pregnancy test performed at least 36 weeks before surgery (Positive serum pregnancy test, performed at least 36 weeks before the date of cesarean delivery, can be used to verify gestational age.) E) Ultrasound examination between 6 and 11 weeks of pregnancy (An early ultrasound is one of the most reliable and accurate ways to date a pregnancy.)

During which phase of the endometrial cycle does ovulation occur? A) Menstrual phase B) Proliferative phase C) Secretory phase D) Ischemic phase

C) Secretory phase (The secretory phase extends from the day of ovulation to about 3 days before the next menstrual period.)

A nurse is concerned that her patient is at risk for sepsis. Which findings in the patient's assessment rule out the diagnosis of sepsis? A) Pulse = 110 beats/min B) 20-mL sample of dark-yellow urine C) T = 97.3°F (36.3°C) D) BP = 100/65 E) RR = 18 and lungs clear bilaterally

C) T = 97.3°F (36.3°C) (A septic patient will oftentimes have a fever of greater than 101°F (38°C). A temperature within normal limits would rule out the concern for sepsis.) D) BP = 100/65 (A septic patient will oftentimes have a low BP, especially if that patient is going into septic shock. A BP within normal limits would rule out the concern for sepsis.) E) RR = 18 and lungs clear bilaterally (A septic patient will oftentimes have shallow and rapid breathing with a rate > 20 breaths/min. A RR rate within normal limits would rule out the concern for sepsis.)

Which action would the nurse take for a breastfeeding mother who is afebrile but reports erythema and tenderness over her right breast? A) Apply cold compress. B) Administer local anesthetic. C) Teach breast massage technique. D) Administer intramuscular antibiotic.

C) Teach breast massage technique. (The patient's symptoms indicate a plugged mammary duct. The nurse would teach the mother to massage the breast to relieve pain and unplug the milk duct.)

Which statement describes how the endometrium responds to the female reproductive cycle? A) The myometrium causes blood loss during menarche. B) The functional layer of the endometrium releases estrogen and progesterone. C) The functional layer of the endometrium is shed during the menstrual period. D) The basal layer of the endometrium regenerates the myometrium each month.

C) The functional layer of the endometrium is shed during the menstrual period. (The functional layer of the endometrium is shed during each menstrual period and is responsible for blood loss during menarche.)

Which statements are true regarding rubella in pregnancy? A) Women develop a "slapped-cheek rash." B) Titers less than 1:8 demonstrate immunity. C) The virus can cross the placental barrier. D) Most women of childbearing age in the United States have received this vaccine already. E) If a woman is not immune, she can be offered the vaccine during pregnancy.

C) The virus can cross the placental barrier. (Crossing of the placental barrier is a serious concern with rubella infection.) D) Most women of childbearing age in the United States have received this vaccine already. (The measles-mumps-rubella (MMR) vaccine contains the rubella vaccine.)

The nurse receives reports on two women in early labor, a nulliparous woman and a multiparous woman. Both are 3 cm dilated. Which statement is true regarding who will enter active labor first? A) The multiparous woman will enter active labor first. B) The nulliparous woman will enter active labor first. C) They will both progress at similar rates. D) The patient whose contractions palpate stronger will enter active labor first.

C) They will both progress at similar rates. (Nulliparous and multiparous women have been found to progress at similar rates in this phase.)

For which reason would a nurse administer a narcotic to a woman at the beginning of a contraction? A) To allow the medication to be transferred to both the laboring woman and the fetus B) To allow for no medication to be transferred to the fetus C) To allow for less medication to be transferred to the fetus D) To decrease the likelihood of maternal hypotension

C) To allow for less medication to be transferred to the fetus (To obtain the most beneficial effects of the opioid analgesia during labor and limit transfer to the fetus, the nurse would start the injection of the medication at the beginning of the contraction, when the blood flow to the placenta is normally reduced. When placental blood flow resumes, much of the drug is in maternal tissues (El-Wahab & Fernando, 2014).)

A patient in early pregnancy presents with heavy bleeding and blood clots. Her hemoglobin and hematocrit are low. Based upon the information given, which order would the nurse anticipate? A) Prepare for emergency cesarean birth. B) Administer 650 mg acetaminophen for pain stat. C) Transfuse 1 unit packed red blood cells stat. D) Massage the fundus.

C) Transfuse 1 unit packed red blood cells stat. (When hemoglobin and hematocrit are low as a result of hemorrhage, blood transfusion is necessary to replace blood lost.)

Which assessment finding is most concerning for a fertility specialist who is working with a 38-year-old couple who has practiced unprotected intercourse for the past 5 years? A) The man occasionally withdraws his penis during ejaculation B) Spontaneous abortion 3 years ago at 8 weeks' gestation C) Unprotected intercourse for 5 years D) Vegetarian diet

C) Unprotected intercourse for 5 years (Unprotected intercourse for 5 years, combined with the age of the couple, is the most concerning assessment finding for the health care provider.)

The nurse caring for the woman in labor recognizes that maternal hypotension puts the laboring woman at risk for which result? A) Early decelerations B) Fetal dysrhythmias C) Uteroplacental insufficiency D) Spontaneous rupture of membranes

C) Uteroplacental insufficiency (Low maternal blood pressure reduces placental blood flow during uterine contractions and results in fetal hypoxemia. This hypoxemia can lead to late decelerations in the fetal heart rate.)

On assessment, the postpartum nurse notes a firm fundus, bright red blood oozing from the vagina, and a saturated perineal pad. What diagnosis would the nurse expect based on these assessment findings? A) Vaginal hematoma B) Placenta accreta C) Vaginal laceration D) Uterine inversion

C) Vaginal laceration (A vaginal laceration that was not discovered or repaired after a vaginal delivery would create an oozing of blood that is typically bright red (frank) in color.)

On assessment, the nurse learns that a male toddler born with a congenital heart defect lives with his parents and two young siblings in the Midwest. The mother is pregnant with a fourth child. Which additional information would the nurse need to know to evaluate the risk for the fourth child being born with a heart defect? A) Socioeconomic status of the family B) The mother's family history of tobacco use C) Variations of symptoms because of time or season D) Sex of family members born with a heart defect E) Number of close relatives born with a heart defect

C) Variations of symptoms because of time or season (Seasonal variations of symptoms are a factor that may influence the risk for developing a heart defect. The nurse would need this information to evaluate the fourth child's risk for developing a heart defect.) D) Sex of family members born with a heart defect (Sex of affected family members is a factor that may influence the risk for developing a heart defect, as it can help determine whether the trait is X-linked. The nurse would need this information to evaluate the fourth child's risk for developing a heart defect.) E) Number of close relatives born with a heart defect (Number of affected relatives is a factor that may influence the risk for developing a heart defect. The nurse would need this information to evaluate the fourth child's risk for developing a heart defect.)

When teaching about infant feeding cues in the first days of life, which solution can the nurse consider to encourage the best time to initiate breastfeeding? A)Whenever the newborn begins to cry B) Every 2 hours, around the clock C) When the infant is in the quiet alert state D) When the newborn naturally wakes itself up, regardless of the time interval

C) When the infant is in the quiet alert state (During the quiet alert state, the newborn is observing the world, and the newborn is calm and relaxed. This is an optimal time to initiate a feeding.)

Match the expected event with the stage of labor. Cervical effacement and dilatation from onset of true labor to complete dilatation; Complete cervical dilatation and effacement to birth of the baby; Birth of the baby to expulsion of the placenta; Expulsion of the placenta to physical recovery of mother and infant

Cervical effacement and dilatation from onset of true labor to complete dilatation: First stage Complete cervical dilatation and effacement to birth of the baby: Second stage Birth of the baby to expulsion of the placenta: Third stage Expulsion of the placenta to physical recovery of mother and infant: Fourth stage

A pregnant woman has several screenings and assessments completed during pregnancy. Which is the correct order? 1-hour glucose tolerance test; Chlamydia culture; Group B streptococcus (GBS) culture; 3-hour glucose tolerance test

Chlamydia culture 1-hour glucose tolerance test 3-hour glucose tolerance test Group B streptococcus (GBS) culture

Match the contraception method with the manner in which it can prevent pregnancy. Creates a physical barrier that prevents the sperm from reaching the egg Creates a spermicidal environment through a sterile, inflammatory response Delays or inhibits ovulation Prevents sperm from entering semen during ejaculation Vasectomy; Intrauterine device (IUD);Emergency contraception (EC); Male condom

Creates a physical barrier that prevents the sperm from reaching the egg: Male condom Creates a spermicidal environment through a sterile, inflammatory response: Intrauterine device (IUD) Delays or inhibits ovulation: Emergency contraception (EC) Prevents sperm from entering semen during ejaculation: Vasectomy

During fundal assessment 30 minutes after a cesarean delivery, the patient reports discomfort and exhaustion and asks the nurse, "Why do you keep poking at me?" Which response from the nurse is appropriate? A) "I'll wait until you're on the postpartum floor to assess you again." B) "I need to do this assessment to make sure your incision is healing correctly." C) "This assessment must be done every 30 minutes until you transfer out of the recovery room." D) "A cesarean delivery puts you at risk for excessive bleeding, so it's important to assess you frequently."

D) "A cesarean delivery puts you at risk for excessive bleeding, so it's important to assess you frequently." (Fundal assessment must be performed frequently after cesarean delivery to assess for uterine atony, which can lead to hemorrhage. Postpartum hemorrhage and uterine atony are risks of cesarean delivery.)

To assess whether there is any improvement in a patient's dysuria, the nurse would ask which question? A) "Do you have to urinate at night?" B) "Do you have blood in your urine?" C) "Do you have to urinate frequently?" D) "Do you have pain when you urinate?"

D) "Do you have pain when you urinate?" (To assess for dysuria, the nurse would ask about pain when urinating.)

The nurse is caring for a patient who has just received genetic counseling and is visibly distressed. The patient tearfully explains to the nurse that the counselor informed her there was a 50% chance of her child having cystic fibrosis and asks, "What should I do about my pregnancy?" Which response from the nurse is appropriate for this patient? A) "Did the counselor say if there was anything you could do to lower the chance?" B) "We can get you information about adoption, if you think you might want to look at it." C) "A 50% chance isn't very high. There's an equal chance your baby will be perfectly healthy." D) "I know you're faced with a difficult decision, but I'm here to support the choices you make."

D) "I know you're faced with a difficult decision, but I'm here to support the choices you make." (Genetic counseling is nondirective. The patient's autonomy should be respected, even when the patient does not know what to do. This response is appropriate because it acknowledges the patient's distress but does not make a decision for the patient.)

Which statement by a postpartum woman indicates that the teaching by the nurse regarding thrombus formation was successful? A) "I'll stay in bed for the first 3 days after my baby is born." B) "I'll keep my arms elevated with pillows." C) "I'll sit in my rocking chair most of the time." D) "I'll put my support stockings on every morning before rising."

D) "I'll put my support stockings on every morning before rising." (Venous congestion begins as soon as the woman stands up. The stockings should be applied before she rises from the bed in the morning.)

A woman who is at 38 weeks gestation reports suspected signs of labor to the triage nurse. Which statement by the nurse supports the beginning of true labor? A) "Your contractions will decrease with activity." B) "The contractions will be mild and more annoying than painful." C) "You will feel the contractions in your front pelvic area and not in your back." D) "Labor contractions will occur in a consistent pattern that increases in frequency, duration, and intensity."

D) "Labor contractions will occur in a consistent pattern that increases in frequency, duration, and intensity." (Contractions occurring in a consistent pattern of increasing frequency, duration, and intensity are a sign of true labor.)

A patient with no family history of genetic abnormalities asks the nurse if there may still be a need for prenatal genetic counseling. Which patient statement indicates that genetic counseling may be necessary? A) "My partner is 14 years older than me." B) "I was adopted by my biological mother's sister." C) "One of my children went deaf in one ear after an ear infection. D) "My family and I practice a religion that does not allow marriage outside of our religion."

D) "My family and I practice a religion that does not allow marriage outside of our religion." (Patients who are members of groups that are isolated by geography, religion, or culture are at increased risk for autosomal recessive traits.)

A patient with gestational diabetes experiences rupture of membranes at 36 weeks gestation. Which patient teaching statement regarding this condition is accurate? A) "Rupture of membranes is more common in women with gestational diabetes because of the excess amounts of glucose in your urine." B) "Rupture of membranes at this time in your pregnancy is normal and sometimes happens before labor begins." C) "Your membranes likely ruptured because of excess ketones in your blood." D) "Preterm rupture of membranes is more common in women with gestational diabetes, and it might be related to overdistention of your uterus."

D) "Preterm rupture of membranes is more common in women with gestational diabetes, and it might be related to overdistention of your uterus." (It is thought that the cause of increased incidence of preterm rupture of membranes in gestational diabetics is overdistention of the uterus.)

A woman admitted at 30 weeks gestation for preterm labor asks the nurse, "How important is surfactant for my baby's lungs, and when is it produced?" Which statement by the nurse best explains when and why surfactant is secreted? A) "Surfactant is first produced in the third trimester to help the baby breathe in utero." B) "Surfactant is produced for alveoli development when the lungs start developing in the first trimester." C) "Surfactant is initially produced at the beginning of the second trimester to keep the lungs from collapsing." D) "Surfactant is produced near the end of the second trimester to help the baby survive but it isn't used in utero by the lungs."

D) "Surfactant is produced near the end of the second trimester to help the baby survive but it isn't used in utero by the lungs." (Surfactant is produced in the lungs to reduce the surface tension of fluid and help make the lung air sac more stable after birth.)

Which educational information regarding breast care would the nurse provide for a mother who chooses to bottle-feed her infant? A) "Apply a hot compress to the breasts for comfort." B) "Pump the breasts if they become engorged after birth." C) "Clean breasts with hot water and soap in the shower." D) "Take analgesics as needed for engorgement discomfort." E) "Apply ice packs to the breasts for several days."

D) "Take analgesics as needed for engorgement discomfort." (Analgesics will decrease inflammation and pain related to breast engorgement.) E) "Apply ice packs to the breasts for several days." (Cold compresses will decrease inflammation and pain related to breast engorgement.)

Which statement by a maternal home care nurse reflects the correct approach to addressing potential and actual postpartum depression in maternal patients? A) "We include education about postpartum depression to women if they have a history of depression." B) "If we suspect a woman may have developed postpartum depression, then we provide specialized education about that topic." C) "Because emotional disorders and imbalances are a very sensitive subject, we try not to offend patients by routinely bringing up the topic of postpartum depression." D) "Teaching about postpartum depression is a routine part of education for all pregnant patients."

D) "Teaching about postpartum depression is a routine part of education for all pregnant patients." (The Centers for Disease Control and Prevention recommends that health care providers address the issue of postpartum depression with all pregnant patients during prenatal visits, preferably during the third trimester, to recognize any potential risk factors for postpartum depression and to begin education immediately on prevention and treatment.)

The student nurse knows that during the latter part of the first trimester, the intestines descend into the abdomen. Which statement provides the best explanation for this physiologic change? A) "The intestines shrink, allowing descent to occur." B) "The umbilical cord expels the intestines into the abdominal cavity." C) "The kidneys and the liver must shrink to allow room for the intestines." D) "The abdominal cavity increases in size, allowing the intestines to descend."

D) "The abdominal cavity increases in size, allowing the intestines to descend." (By week 10, the abdominal cavity is proportional in size compared with the rest of the body and can accommodate the intestines.)

Which statements by the nurse are the most appropriate to make to a patient who has recently had a miscarriage? A) "When it is meant to happen, it will happen." B) "The fetus likely had a congenital anomaly that was incompatible with life." C) "Don't worry; nature has a way of taking care of mistakes. It is for the best." D) "This is often a consequence of unavoidable factors, and it is important to realize this is not your fault." E) "I am very sorry for your loss. Having a miscarriage must be so emotionally painful. How are you feeling?"

D) "This is often a consequence of unavoidable factors, and it is important to realize this is not your fault." (The nurse can help the patient by emphasizing that most spontaneous abortions are caused by unavoidable factors or abnormalities.) E) "I am very sorry for your loss. Having a miscarriage must be so emotionally painful. How are you feeling?" (Acknowledging the patient's pregnancy loss and associated emotional pain is part of supportive nursing care.)

After a primipara's admission to the labor and delivery suite, the nurse assesses her discharge needs. She will be discharged home 4 days after a cesarean delivery. Which questions would the nurse ask the patient? A) "Have you ever had anesthesia before?" B) "Do you have any allergies to shellfish?" C) "Do you have a name picked out?" D) "Will you have help when you go home?" E) "Are there many stairs in your home?"

D) "Will you have help when you go home?" (To determine the discharge needs of the primipara, it is important for the nurse to know if the patient will be returning home with or without help from a partner or family member. Patients who live alone or do not have access to help after a cesarean delivery may need special accommodations, including a home-care visit from a postpartum nurse.) E) "Are there many stairs in your home?" (To determine the discharge needs of the primipara after a cesarean delivery, it is important for the nurse to know if the patient will have to maneuver flights of steps. This patient will need additional education about how much activity is appropriate following a cesarean delivery.)

Which education would the nurse provide the mother of a newborn and 2-year-old who expresses concern about the baby's sleep schedule? A) "You should set a strict schedule for your baby's naps." B) "During active sleep, if the baby moves, she is waking up." C) "Feeding your baby during the drowsy state will help promote sleep." D) "Your baby will not be disturbed by other children during quiet sleep."

D) "Your baby will not be disturbed by other children during quiet sleep." (During quiet sleep, the newborn is not disturbed by noise.)

Which finding would the nurse recognize as indicating progression of preeclampsia to eclampsia? A) Proteinuria B) Weight gain in the absence of edema C) Pitting edema in the absence of weight gain D) A generalized seizure not attributed to other causes

D) A generalized seizure not attributed to other causes (Generalized seizure is the defining sign of eclampsia.)

The nurse recognizes that a steady trickle of bright red blood from the vagina in the presence of a firm fundus may indicate which condition? A) Uterine atony B) Retained placenta C) Infection of the uterus D) A laceration within the genital tract

D) A laceration within the genital tract (Undetected lacerations bleed slowly and continuously until repaired with sutures. Bleeding from lacerations is uncontrolled by uterine contractions, and it is typically bright red.)

Which patient would most likely require general anesthesia during labor and delivery? A) A patient with a vaginal infection B) A patient with severe pain from uterine contractions C) A patient requiring a vaginal episiotomy D) A patient refusing regional and local anesthetic methods who needs a cesarean delivery

D) A patient refusing regional and local anesthetic methods who needs a cesarean delivery (A patient who refuses regional and local anesthetic methods and needs a cesarean delivery does require general anesthesia.)

Maternity nurses often have to answer questions about the many, sometimes unusual, ways people have tried to make the birthing experience more comfortable. Which information would the nurse consider when educating patients and support people about nonpharmacologic pain management techniques? A) Music supplied by the support person has to be discouraged because it could disturb others or upset the hospital routine. B) Women in labor can benefit from sitting in a bathtub, but they must limit immersion to no longer than 15 minutes at a time. C) Effleurage is permissible, but counterpressure is almost always counterproductive. D) Acupressure to either side of the spine can facilitate the release of endorphins and decrease some back pain.

D) Acupressure to either side of the spine can facilitate the release of endorphins and decrease some back pain. (Acupressure is an effective nonpharmacologic technique that can be used to relieve a variety of pain during labor.)

Which location for a superficial venous thrombosis is the most common? A) Top of the foot B) Across the knees C) Groin area D) Behind the knees

D) Behind the knees (Any superficial vein may become varicosed, but the veins most commonly affected are those in the legs, especially behind the knees.)

Which nursing finding requires intervention when assessing a postpartum woman who delivered by cesarean birth? A) A small amount of lochia B) Foley catheter output of 200 mL/hour C) Abdominal dressing with a small amount of serosanguinous drainage D) Distended abdomen with no bowel sounds auscultated

D) Distended abdomen with no bowel sounds auscultated (Abdominal distention and the absence of bowel sounds would concern the nurse. This may indicate the presence of bowel obstruction.)

A patient reports a feeling of lightness in her abdomen, increasing whitish, thin, and odorless vaginal discharge, and occasional contractions that go away after an hour or so. Which action would the nurse take next? A) Advise the patient to report to the birthing center. B) Suggest routine sexual transmitted infection counseling. C) Call the ambulance because she is experiencing an emergency. D) Educate the patient on the premonitory signs of labor.

D) Educate the patient on the premonitory signs of labor. (The nurse would educate the patient on premonitory signs of labor and counsel the patient that what she is experiencing is normal. The nurse would also educate on true versus false labor signs.)

Which nursing intervention can help reduce a family's stress related to adaptation after birth? A) Instruct parents about neonatal cues. B) Provide extra attention to siblings that visit. C) Provide opportunities for the parents to perform newborn care. D) Encourage the parents to identify social support to assist them after discharge.

D) Encourage the parents to identify social support to assist them after discharge. (Encouraging the parents to identify social support from friends, family, and support groups within the community will help decrease stress related to adaptation after birth.)

Which fetal heart rate finding may result from maternal fever? A) No expected changes B) Prolonged decelerations C) Fetal heart rate of 90 beats/min for 15 minutes D) Fetal heart rate of 180 beats/min for 12 minutes

D) Fetal heart rate of 180 beats/min for 12 minutes (A maternal fever can directly increase the fetal temperature or infect the fetus in cases of infection. The fetus responds with an increased heart rate, which can lead to fetal tachycardia.)

A patient is experiencing intense back pain and discomfort during active labor. Which position would the nurse encourage the patient to assume? A) Standing straight B) Side-lying C) Semi-sitting D) Hands and knees

D) Hands and knees (A hands-and-knees position reduces back pain because the fetus falls forward, away from the sacral promontory. It promotes normal mechanisms of birth. The woman can use pelvic rocking to decrease back pain. Caregivers can rub the woman's back or apply sacral pressure easily.)

Which uterine contraction strength classification is used when labor is measured at >500 Montevideo units (MVUs)? A) Normal B) Moderate C) Hypotonic D) Hypertonic.

D) Hypertonic (Above 400 MVUs, the uterine contraction strength is considered hypertonic, which is abnormal.)

Which physiologic change in a pregnant woman is likely responsible for frequent vaginal yeast infections? A) Increased estrogen levels B) Increased prolactin levels C) Decreased progesterone levels D) Increased glycogen secretion within the vaginal walls

D) Increased glycogen secretion within the vaginal walls (Increased glycogen secretion from the vaginal walls favors the growth of yeast infections during pregnancy.)

Which statement explains how intrathoracic pressure contributes to newborn respiratory adaptation? A) It activates chemoreceptors in the carotid arteries and aorta. B) It activates skin sensors that send impulses to stimulate the respiratory center. C) It causes a drop in prostaglandin levels, thereby reducing inhibition of respirations. D) It forces fetal lung fluid out of the alveoli and into interstitial spaces.

D) It forces fetal lung fluid out of the alveoli and into interstitial spaces. (Pressure changes in the lungs force fetal lung fluid out of the alveoli to allow air into the lungs.)

In which way is the hypothalamus responsible for sexual maturation? A) It produces estrogen to stimulate growth of breast tissue. B) It releases luteinizing hormone (LH) to stimulate growth of the ovaries. C) It produces follicle-stimulating hormone (FSH) to stimulate maturation of the follicles. D) It releases gonadotropin-releasing hormone (GnRH) to stimulate the anterior pituitary gland.

D) It releases gonadotropin-releasing hormone (GnRH) to stimulate the anterior pituitary gland. (The hypothalamus is responsible for sexual maturation by releasing GnRH to stimulate the anterior pituitary gland to release FSH and LH.)

After administration of oral (PO) labetalol for a patient with gestational hypertension, the nurse notes a blood pressure (BP) of 178/96 mm Hg. The patient also reports headache and blurry vision. Which order from the patient's health care provider would the nurse anticipate? A) Up ad lib B) Lisinopril C) Salt-free diet D) Magnesium sulfate

D) Magnesium sulfate (Headache, blurred vision, and a BP of 178/96 mm Hg are indicative of preeclampsia. Magnesium sulfate would be administered to prevent seizures.)

Which assessment finding explains why a 25-year-old woman reports anovulation for the past 1.5 years? A) Breast tenderness B) Previous ectopic pregnancy C) Small fibroid tumor D) Marathon training

D) Marathon training (Strenuous or intense exercise, such as marathon training, may cause ovulatory dysfunction and secondary anovulation.)

What are the four phases of the endometrial cycle that describe changes to the endometrium in the correct order? A) Proliferative, secretory, ischemic, and menstrual phases B) Proliferative, menstrual, secretory, and ischemic phases C) Menstrual, secretory, proliferative, and ischemic phases D) Menstrual, proliferative, secretory, and ischemic phases

D) Menstrual, proliferative, secretory, and ischemic phases (After the endometrium is shed (menstrual phase), there is rapid growth of the endometrium (proliferative phase). Then there is a 3-day period after ovulation (secretory phase), and if no fertilized ovum is implanted, the endometrium separates, and bleeding occurs (ischemic phase).)

Which nursing instruction is appropriate when recommending food or drink to enhance the absorption of iron during pregnancy? A) Milk B) Water C) Cheese D) Orange juice

D) Orange juice (Vitamin C-enriched fluids/foods, such as orange juice, enhance the absorption of iron.)

Which action would the nurse take to improve placental blood flow immediately after administration of an epidural? A) Place the woman flat on her back B) Place the woman prone C) Place the woman in the high-Fowler position D) Place a wedge under the woman's right hip

D) Place a wedge under the woman's right hip (Placing a wedge under the right hip improves placental blood flow after a procedure that causes vasodilation.)

When assessing the neonate born to a mother with preeclampsia, the nurse knows that low birth weight may be attributed to which factor? A) Maternal diabetes B) Limited maternal diet C) Maternal hypotension D) Poor placental perfusion

D) Poor placental perfusion (Decreased blood flow to the placenta leads to decreased nutrients to the fetus and can impair fetal growth.)

Which nonreassuring fetal heart rate (FHR) pattern includes a reduction in FHR of ≥15 beats/min for more than 2 minutes? A) Accelerations B) Tachycardia C) Early decelerations D) Prolonged decelerations

D) Prolonged decelerations (Prolonged decelerations cause a decrease in FHR of ≥15 beats/min below baseline and last longer than 2 minutes but less than 10 minutes. The return to baseline FHR usually occurs after the contraction is over.)

A patient experiences nausea and vomiting during labor. Which medication might the nurse anticipate administering? A) Fentanyl B) Naloxone C) Nalbuphine D) Promethazine

D) Promethazine (The nurse would anticipate promethazine because it relieves the nausea and vomiting that may occur when opioid drugs are given.)

A woman in labor is given an opioid analgesic for pain. Which vital sign is a priority for the nurse to monitor in the newborn infant? A) Pulse B) Temperature C) Blood pressure D) Respiratory rate

D) Respiratory rate (Opioid analgesics can cause respiratory depression, which is more likely to occur in the newborn than in the mother; therefore respiratory rate should be monitored.)

Which maternal-infant blood type combination may lead to maternal sensitization during pregnancy? A) Maternal blood type AB; fetal blood type B B) Rh-positive mother; Rh-positive fetus C) Maternal blood type O; fetal blood type A D) Rh-negative mother; Rh-positive fetus

D) Rh-negative mother; Rh-positive fetus (Maternal sensitization is related to Rh incompatibility, which occurs with an Rh-negative mother and Rh-positive fetus.)

Which food or beverage is the most important for a pregnant woman at 32 weeks of gestation to consume before, during, and after her tennis matches? A) Extra protein sources, such as peanut butter crackers B) Salty foods to replace lost sodium C) Caffeinated energy water D) Several glasses of water

D) Several glasses of water (If no medical or obstetric problems contraindicate physical activity, pregnant women should get 30 minutes of moderate physical exercise daily. Liberal amounts of fluid should be consumed before, during, and after exercise because dehydration can trigger premature labor.)

Which nursing instruction is appropriate to give a pregnant woman who will be travelling in a car for 8 hours? A) Decrease fluid intake. B) Drive with minimal stops. C) Avoid travel by automobile. D) Stop frequently to walk.

D) Stop frequently to walk. (The nurse should instruct the patient to stop every 2 hours and ambulate for 10 minutes. This will help prevent the formation of clots in the lower extremities.)

Which maternal or fetal condition might require an amnioinfusion? A) The mother has polyhydramnios (excess amniotic fluid). B) The fetal heart rate (FHR) is 140 beats/min. C) The patient's blood pressure has dropped. D) The fetus has experienced umbilical cord compression.

D) The fetus has experienced umbilical cord compression. (One common indication for amnioinfusion is umbilical cord compression. Instillation of sterile fluid often relieves the compression.)

A pregnant woman arrives at the emergency department, and after completing a vaginal examination the nurse midwife states that the patient is 5 cm dilated and 75% effaced and the fetus is at −3 station. Which statement is accurate regarding this assessment? A) The patient may initiate pushing efforts. B) The cervix is fully dilated. C) The cervix is fully effaced. D) The fetus is not engaged within the maternal pelvis.

D) The fetus is not engaged within the maternal pelvis. (Fetal engagement begins at 0 station.)

When assessing a patient with preeclampsia on magnesium sulfate for seizure prophylaxis, the nurse notes the patient demonstrates hyperreflexia (+4). How does the nurse interpret this patient cue? A) The nurse should administer a 500-mL bolus of lactated Ringer's solution. B) The nurse should interpret this as normal and continue the therapy as prescribed. C) The nurse should request a prescription for calcium gluconate immediately. D) The nurse should question the health care provider regarding the current dose of magnesium sulfate.

D) The nurse should question the health care provider regarding the current dose of magnesium sulfate. (Hyperreflexia and neurologic instability are signs of worsening preeclampsia. This patient's magnesium sulfate is likely subtherapeutic, and the health care provider should prescribe the rate to be increased to prevent seizure.)

When administering corticosteroids to a patient in preterm labor, which information is relevant to the nurse regarding the patient's history? A) The patient is 33 weeks' gestation. B) The patient suffers from chronic hypertension. C) The patient's membranes are ruptured. D) The patient has type 1 diabetes.

D) The patient has type 1 diabetes. (Corticosteroids can increase blood glucose levels and increase insulin resistance; therefore they should be administered cautiously or avoided altogether in patients with type 1 diabetes to avoid complications.)

Which reasoning is appropriate when documenting reported fetal movement as a presumptive sign of pregnancy? A) The symptoms have no alternative causes. B) The symptoms could have alternative causes such as uterine tumors, polyps, or ascites. C) The symptoms are associated with a "woman's intuition." D) The symptoms could have alternative causes such as peristalsis, flatus, or abdominal muscle contractions.

D) The symptoms could have alternative causes such as peristalsis, flatus, or abdominal muscle contractions. (Quickening is a presumptive sign of pregnancy that typically starts around 16 to 20 weeks' gestation.)

Which situation demonstrates how an environmental factor could negatively affect a woman's experience of pain during labor? A) The woman experienced tearing of the perineum during delivery. B) The woman had a complicated delivery with her previous child. C) The woman's family views vocalizing about pain as "complaining." D) The woman has no support at her bedside.

D) The woman has no support at her bedside. (Environmental factors that affect a woman's perception of pain include a support person or people present, a supportive physical environment, and comfort with her health care providers.)

Which phrase describes the purpose of the luteal phase of the female reproductive cycle? A) To release the mature ovum B) To promote the growth of the follicle C) To enable menstrual flow D) To prepare the endometrium for the ovum

D) To prepare the endometrium for the ovum (During the luteal phase, the corpus luteum secretes estrogen and progesterone to prepare the endometrium for a fertilized ovum.)

Match the method of heat loss with the nursing action that would prevent it. Keep newborn dry and covered. Place a blanket between the newborn and the examination table. Move the crib away from air vents. Move the crib away from the window or outside wall. Conduction, Evaporation, Radiation, Convection

Keep newborn dry and covered: Evaporation Place a blanket between the newborn and the examination table: Conduction Move the crib away from air vents: Convection Move the crib away from the window or outside wall: Radiation

Match the type of diabetes with its characterization. Lack of insulin secretion Insulin resistance Early gluconeogenesis Gestational diabetes mellitus; Type 1 diabetes mellitus; Type 2 diabetes mellitus

Lack of insulin secretion: Type 1 diabetes mellitus Insulin resistance: Type 2 diabetes mellitus Early gluconeogenesis: Gestational diabetes mellitus

Match the female reproductive structure with its role during childbirth. Muscle contracts to expel fetus. Fibrous tissue opens to accommodate fetus's passage into the birth canal. Muscle expands as fetus moves through birth canal. Muscle stretches as fetal head emerges. Perineum; Uterus; Vagina; Cervix

Muscle contracts to expel fetus: Uterus Fibrous tissue opens to accommodate fetus's passage into the birth canal: Cervix Muscle expands as fetus moves through birth canal: Vagina Muscle stretches as fetal head emerges: Perineum

Match the appropriate type of anesthesia to the patient description. Patient requires a vaginal laceration repair after delivery Patient in labor wants pain relief while being awake for the birth Patient with active skin infection requires an emergency cesarean delivery Patient wants pain relief while being able to move and feel contractions Combined spinal-epidural anesthesia; Epidural block; Pudendal block; General anesthesia

Patient requires a vaginal laceration repair after delivery: Pudendal block Patient in labor wants pain relief while being awake for the birth: Epidural block Patient with active skin infection requires an emergency cesarean delivery: General anesthesia Patient wants pain relief while being able to move and feel contractions: Combined spinal-epidural anesthesia

n which direction does neonate blood flow from the inferior vena cava? Place the structures in the order of blood flow from the inferior vena cava in the neonate. Pulmonary artery; Right Atrium; Lungs; Right Ventricle

Right atrium Right ventricle Pulmonary artery Lungs This is the circulation process of a normal human heart. Deoxygenated blood enters the right atrium and then travels through the tricuspid valve during diastole into the right ventricle. During systole, the heart pumps blood from the right ventricle into the pulmonary artery, which delivers deoxygenated blood to the lungs to allow for adequate oxygenation.


Related study sets

Disease of the Nasal Cavity, Sinuses, Pharynx, and Larynx

View Set

Henri IV and the Edict of Nantes.

View Set

Developmental chapter 17, 18, 19, 20

View Set